Any Flashcards
Un-categorised Vivas
What is an abscess?
Localised collection of pus in an enclosed tissue space.
Aetiology:
- Pyogenic bacteria: Staphylococcus aureus, Streptococcus pyogenes and E.coli
- Reaction foreign body e.g. needle, splinter
Pathogenesis:
- Pathogens or foreign material cause localised cell damage and release of cytokines.
- This triggers an inflammatory response attracting WBCs, particularly neutrophils.
- There is a build-up of pus which is a mixture containing cellular debris, WBCs and bacteria.
- Simultaneously, nearby cells form a capsule of tissue around the pus to prevent it from spreading.
PC:
Superficial (e.g. skin):
- Signs of inflammation: Redness, swelling, pain, heat
- Fluctuant mass
Deep (e.g. liver, brain, renal):
- Systemic symptoms: Fever, anorexia, weight loss, fatigue
- Abnormal organ function
Mx:
Incision and drainiage
- Superficial: Via scalpel
- Deep: Percutaneously via US/CT guidance
- Abx usually not indicated as it can’t perforate core of abscess. If perforated, consider empirical Abx (Varies on location, refer to local guidelines).
Examples:
- Skin abscess
- Liver abscess following portal vein pyema related to bowel leakage/peritonitis or billiary infection
- Renal/perinephric abscess usually complicate urologic infection due to G-ve enteric bacilli or secondary haematogenous seeding
What is cancer staging?
System used to classify cancers to provide prognostic information and guide mx.
TNM Staging:
- T: Extent of tumor invasion. Tx = cannot be found. T0 = occult (can’t be found). T1-4 = based of size and local invasion.
- N: Nodal involvement. Nx = cannot be measured. N0 = no nodal involvement. N1-3 = stage increases with increasing number of nodes, presence of large/matted nodes, or nodes more distant from the primary tumor.
- M: Mx = cannot be measured. M0 = cancer has not metastasized. M1 = metastatic disease present (usually implying haematogenous spread).
Note there are some malignancies that do not use TNM e.g. Gynaecological with FIGO system, Lymphoma with the Arbor system, Colon cancer which uses the dukes classification system.
Tumor Grading:
Different to staging and and provides quantification of cellular changes of cancer cells, assessed by microscopic analysis. Grades from X-4.
What is the purpose of “staging” a patient’s cancer?
- Assist with planning of treatment
- Deciding whether aim is curative or palliative
- Type of treatment that is most likely to be successful - Gives an estimate of prognosis
- Provides common terminology in assessing cancers
- Helps with communication between healthcare providers and researchers
- Helps to evaluate and compare clinical trials
What modalities are used to stage a patient’s cancer?
The TNM criteria, explained further in the “What is cancer staging” viva, is often used and there are several modalities are used to determine the extent of tumor size/invasion, nodal involvement and metastatic spread including:
- History and Physical examination:
Assess for signs/symptoms of cancer and/or metastatic disease (e.g. weight loss, fever, night sweats). - Imaging:
Structural (X-ray, CT, MRI, Scopes)
Detects the presence of a tumor and to an extent it’s size/shape.
To increase sensitivity a contrast agent may be used.
Functional (e.g. PET scan, (18-FDG), Bone scan (Tc-99m)
- Detect presence of metastatic disease bone and soft tissue.
- False positives may result from infection, inflammation, or granulomatous disease.
- Not useful in cancers with a low metabolic rate (e.g. prostate cancer that have a low uptake).
Biochemistry:
- Tumor markers (e.g. Colorectal cancer – CEA, Pancreatic cancer – Ca125)
- Physiological changes that tumor may cause (e.g. Liver Ca – deranged LFTs)
Histopathology:
- Tumor biopsy or from tissue that has been surgically resected.
- Grading: assess level of cellular differentiation, with a greater differentiation reflecting a faster, more aggressive cancer.
- Lymph nodes to assess for metastatic spread.
What is neoplasia? Give examples of benign and malignant neoplasms.
Neoplasia is new, abnormal growth of tissue.
It involves unchecked cell proliferation due to a combination of: a) genetic and b) environmental exposures. It results in overactive growth pathways and underactive growth suppression pathways.
Classification:
1. Benign:
Abnormal cell growth that lacks the ability to invade local tissue or to metastasize.
Examples (-oma):
- Adenoma (Benign neoplasm of glandular epithelium e.g. Liver/Renal)
- Fibroadenoma (benign neoplasm of breast)
- Leiomyoma (benign neoplasm of smooth muscle)
- Fibromas (fibrous tissue)
- Lipoma (fat tissue)
- Papilloma (finger-like projections emanating from an epithelial surface)
- Polyp (mucous membrane)
- Malignant:
Abnormal cell growth that has the potential to invade surrounding tissue and to metastasize.
Examples (-carcinoma or -sarcoma)
Carcinoma (malignant neoplasm from epithelial tissue)
Adenocarcinoma (malignant neoplasm of glandular tissue)
Melanoma (malignant neoplasm of melanocytes)
Rhabdomyosarcoma (malignant neoplasm of skeletal muscle)
Features:
Benign
- Growth: Slow
- Border: Well-demarcated
- Histology: Well differentiated (similar to tissue of origin)
- Metastases: Absent
Malignant
- Growth: Progressive, eratic (slow to rapid)
- Border: Poorly defined/irregular
- Histology: Poorly differentiated and anaplastic (little/no resemblance to tissue of origin)
- Metastases: Frequently present
What is a fistula? Give some examples.
Definition:
A Fistula is an abnormal anastomosis between two epithelialised surfaces.
Aetiology:
- Congenital
- Injury/surgical
- Infectious
- Inflammation/IBD (e.g. Chron’s, UC)
- Neoplasia
- Radiation
Classification:
- Complete: internal and external openings (i.e. extends from skin to an internal body cavity)
- Incomplete: external (skin) opening and no internal (organ) opening.
Examples:
- Gastrointestinal
- Between organ to another organ e.g. enteroenteral, cholecysto-duodenal, gastrojejunal.
- Between organ and external surface e.g. anal/anorectal, biliary, pancreatic. - Urogenital
- Between vagina and organ (rectum) e.g. rectovaginal (enterovaginal)
- Between bladder and organ e.g. vesicorectal, vesicointestinal, vesicovaginal. - Vascular
- Between artery and vein e.g. arteriovenous fistula - Respiratory
- Between trachea and oesophagus e.g. tracheoesophageal fistula.
Complications: Specific to location, but may result in: - Recurrent infections, sepsis - Bowel or bladder incontinence - Abnormal discharge/bleeding - Pain
What is a stoma?
Natural or artifically created opening between a hollow organ and the external environment.
Examples:
- Natural e.g. mouth, nostrils, anus
- Artificial e.g.
Ileostomy:
Connects ileum with external environment through the anterior abdominal wall (usually right-sided).
Produces large amounts of loose stool, requires changing 4-6x per day.
Colostomy:
Connects the large bowel with the external environment through the anterior abdominal wall (usually left sided).
Produces soft and formed stools with more flatulence, active once per day.
Gastrotomy:
Connects the stomach via tube for nutritional support or gastric decompression.
Tracheostomy:
Surgically created opening in the neck leading directly to the trachea done: a) to bypass an obstructed upper airway, b) to clean and remove secretions from the airway, c) prolonged mechanical ventilation, d) to easily and safely deliver oxygen to the lungs.
Classification:
- Temporary (reversed after 8-10wks)
Rest for distal bowel involved in inflammatory process (e.g. surgery, IBD, abscess)
Emergency relief of distal bowel obstruction (e.g. sigmoid volvulus, colorectal ca)
Protect distal anastomoses
- Permanent
Required when no distal segment is remaining (e.g. resected bowel/anus)
If unable to swallow due to neurological disorder (e.g. cerebral palsy, MS) or oesophageal obstruction (e.g. stricture, atresia, Ca)
Surgical methods:
End:
One lumen, usually colon, brought to abdominal wall and sutured into skin.
Loop:
Loop of bowel is pulled out and sutured onto abdomen and two openings are created for stool and mucous.
Typically used when reversal is planned.
Complications
Early:
-Stoma necrosis, dehiscence or retraction
Late:
- Stenosis, parastomal hernia +/- incarceration
Any time:
- Infection, parastomal bleeding, bowel obstruction, skin irritation, psychosocial impact
Miscellaneous:
- Not painful due to the lack of somatic innervation of gut
- Recommended to eat low fibre diet
What are the potential complications of a surgical procedure?
Anaesthesia:
Anaphylaxis
Infection at cannula or epidural/spinal site
General: Minor trauma to teeth/throat from endotracheal tube Hypotension, MI, stroke Post-operative nausea/vomiting Aspiration +/- pneumonia Malignant hyperthermia
Epidural: (into epidural space) Hypotension Unintentionally high block Spinal: (into cerebrospinal fluid) Headache
Perioperative:
Damage to adjacent structures (haemorrhage, leak of GI contents, sepsis, nerve injury)
Haemorrhage
Laparoscopic to open surgery
Immediate post-operative: (<24hrs)
Basal atelectasis
Haemorrhage
Oliguria (low urine output due to inadequate fluid replacement)
Shock (due to, MI, PE, septicaemia, haemorrhage)
Early post-operative: (24hrs-1m) Pain Infection (SSI, sepsis, pneumonia, UTI) Wound failure (dehiscence) Fluid/electrolyte disturbance (dehydration, fluid overload, hyponatremia, hyperkalaemia) DVT/PE Paralytic ileus Acute renal failure (ATN/nephrotoxins) Urinary retention Delirium Constipation Pressure sores
Late post-operative: (>1m)
Adhesions (bowel obstruction, abdominal pain)
Incisional hernia
Keloid formation
Recurrent infections
Recurrence of reason for surgery (malignancy)
What are the risk factors for a superficial wound infection?
Surgical site infection:
Within 30 days of procedure
Superficial Involves skin and subcutaneous tissue only
At least one of the following features:
Purulent discharge
Positive culture
Evidence of inflammation (pain, erythema, swelling, tenderness, warmth).
Risk factors:
Pre-operative (patient factors):
Immunocompromised state:
Pre-existing infection – HIV/AIDS, carrier of Staphylococcus
Primary immunodeficiencies - CVID
Medications – immunosuppressants, corticosteroids, rituximab
Metabolic – malnutrition, DM, obesity, smoking
Extremes of age
Perioperative:
Type of surgical wound:
Clean: no open viscus, site not inflamed or contaminated
Clean-contaminated: viscus open (GIT, GUT, Resp) but minimal spillage
Contaminated: viscus open with significant spillage
Dirty: active infection with purulent exudate, due to traumatic wound (with retained foreign body or faecal contamination) or ruptured viscus/abscess
Surgical method:
Aseptic technique – surgical scrub, skin prep, sterile instruments, OT environment
Open vs laparoscopic
Duration
ABX prophylaxis
Insertion of surgical drain, prothesis
Type of suture and dressing used
Postoperative:
Glycaemic control
Dressing and cleaning wound
Hygiene practices from health care professionals and visitors
How do you treat a superficial wound infection?
Prevention (optimising risk factors):
Pre-operative: Consider ABX prophylaxis, smoking cessation, medications
Perioperative: Sterile technique, skin preparation with antiseptic (betadine, chlorhexidine)
Post-operative: aseptic technique for regular wound dressings, shower after 48hrs
Mx:
Assess vitals
Wound swabs for culture (+/- blood culture if evidence of systemic involvement or sepsis)
Open wound (remove sutures/clips)
Irrigate and drain pus with 0.9% NaCl
Clean dressings (fluid-absorbent, anti-microbial) with regular changes
Abx: Consider If evidence of spreading cellulitis Ensure wound swab for MCS prior to commencing Pathogens: Staphylococcus Aureus (often MRSA) - Coagulase-negative Staphylococci Gram-negative bacilli Enterococcus Streptococcus species Anaerobes Selection:
Gram positive
Mild: Flucloxacillin 500mg PO, 6-hourly
Gram negative (e.g. post GI/genitourinary) Mild: Amoxycillin + Clavulanate 875 + 125mg PO, 12-hourly
Severe
IV Flucloxacillin + Gentamycin
Suspected MRSA/hypersensitivity to penicillins:
Vancomycin IV
What are the principles of antibiotic prophylaxis for surgical patients?
[NEED TO IMPROVE}
Surgical antimicrobial prophylaxis refers to the use of antibiotics for the prevention of surgical site infections and does not include preoperative decolonisation or treatment of established infections. The principles of antibiotic prophylaxis are:
- Right indication:
The benefit of prophylaxis must be weighed up against the potential risks of antimicrobial use, including allergic reactions, antibiotic associated C.difficile and antibiotic resistance.
Antibiotic prophylaxis is not indicated for clean non-prosthetic procedures, nor for minor surgeries. It is likely to be indicated for procedures where:
a) The incidence of surgical site infection tends to be high, for example colorectal surgery.
b) The consequences of infection are significant, for example surgery with implanted materials such as arthroplasty and cardiac valve surgery. - Right antimicrobial:
a) The choice of antimicrobial is ultimately influenced by the surgical procedure and associated risk factors. It should provide coverage of the expected microbiological flora at the incision site. For the majority of procedures a first generation cephalosporin such as cefazolin (primarily covers gram positive organisms) remains the preferred antimicrobial for prophylaxis. - Right dose and duration:
When indicated, a single dose of antibiotics is sufficient for most procedures. This dose may be influenced by patient related risk factors such as age, renal function and weight.
4. Right route of administration: Parenteral administration (intravenous or intramuscular) is the preferred route for antimicrobial prophylaxis. However there are exceptions including oral antibiotics for transurethral resections of the prostate.
- Right timing of administration:
Most guidelines recommend that antimicrobial prophylaxis is given 60-120 minutes before incision. - Appropriate ABX prophylaxis is required to reduce postoperative infections without increasing antimicrobial resistance.
Why do surgical wounds fail?
Wound dehiscence is the failure of a wound to close properly, includes:
- Simple
- Skin wound alone fails secondary to impaired healing - Burst abdomen
- The separation of abdominal wall closure with protrusion of the abdominal contents due to raised intra-abdominal pressure or from surgical technical failure (poor suturing).
Most common cause is SSI – so essentially anything that causes an infection may cause wound failure.
Risk factors: Pre-operative (anything that leads to immunocompromised state) Increasing age Steroid use Smoking Obesity or malnutrition Chemotherapy/Radiotherapy
Perioperative: Emergency surgery Abdominal surgery Duration >6hrs Poor wound closure (poor suturing, incorrect suture) Poor aseptic technique
Post-operative: Increased pressure: Infection of wound Haematoma Abdominal distension (e.g. cirrhosis) Excessive coughing Heavy lifting Hypoxaemia Prolonged ventilation Poor tissue perfusion (e.g. post-operative hypotension, COPD) Trauma
Clinical features:
Visible opening of wound
Bleeding, discharge (serosanguinous fluid – pinkish colour)
Mx:
Wound site swab and culture
Debride necrotic tissue
Consider prophylactic ABX (indicated ASAP if burst abdomen)
Re-closure
Surgical: Re-suturing the wound using deep retention suture
Non-surgical: saline-soaked gauze packing or negative pressure wound therapy
What is the difference between tissue obtained by fine needle aspiration and a core biopsy?
FINE NEEDLE ASPIRATE
Fine 22G needle aspirates cells/fluid from lesion for cytopathology
Results: Inadequate sample Benign Suspicious Malignant (C1-5)
Advantages: Simple, fast technique On-site interpretation Inexpensive Low risk of complication
Disadvantages:
Requires training in preparation of quality smears
Cancer can be missed if cells not obtained in tissue sample.
Can’t tell if cancer is invasive or not.
CORE BIOPSY
Large bore needle (varying shapes/sizes) collects large tissue sample with architecture preserved for histopathology. Usually performed with guidance.
Results: Similar to FNA Histological tissue types In situ vs invasive Reliable grading Receptor status (breast cancer - ER, PR, HER2)
Advantages:
Higher specificity and sensitivity
Definitive histological diagnosis and gives tissue fragments allowing for architectural features of tissue.
Gives borders and not just cells, allows for assessment of microcalcification in breast cancer.
Disadvantages: Not immediate result Complications (haematoma, haemorrhage, pain, discomfort) More invasive w/ local anaesthetic More expensive
Please look at these arterial blood gas results: pH 7.30, Pa O2 66mmHg, Pa CO2 26mmHg, base excess - 6.5, lactate 9.4. What is a summary description of these results?
Reference ranges for ABGs: pH 7.35-7.45 PaO2 80-100mmHg PaCO2 35-45mmHg HCO3- 22-26mmol/L Base excess -2 to +2 mmol/L Lactate 0.5-2mmol/L
pH 7.3 = 7.3 acidosis PaO2 66 = hypoxaemia PaCO2 26 = low (likely compensatory) Base Excess -6.5 = reduced (metabolic acidosis) Lactate 9.4 = Lactic acidosis (high)
This ABG shows metabolic acidosis with partial respiratory compensation. The low PaO2 and high lactate
suggest cellular hypoxia and subsequent anaerobic metabolism has occurred.
Please look at these arterial blood gas results: pH 7.30, arterial blood gas results: pH 7.30, Pa O2 66mmHg, Pa CO2 26mmHg, base excess - 6.5, lactate 9.4. What causes a blood gas profile of this sort?
Classified into Type A or Type B depending on whether there is impaired tissue perfusion. Biggest clinical concern is a septic patient.
Type A:
Inadequate oxygen delivery triggers anaerobic metabolism, and thus excess lactate as a by-product. This is in response to:
Increased oxygen consumption
E.g. exercise, seizures
Tissue hypoperfusion
E.g. Sepsis, Hypovolemic shock, hypotension, cardiopulmonary arrest, acute HF, mesenteric ischemia
Impaired oxygen-carrying capacity
E.g. anaemia, severe hypoxemia (respiratory failure), carbon monoxide poisoning
Types B:
No impairment of oxygen delivery, but carbohydrate metabolism is disordered resulting in excess lactic acid production.
B1: Underlying disease
E.g. ketoacidosis, malignancy (leukaemia, lymphoma), HIV infection
B2: Iatrogenic/medication
E.g. Alcohol, metformin, anti-retroviral, beta-agonists (epinephrine), paracetamol
B3: Inborn errors of metabolism
Enzyme defects e.g. pyruvate dehydrogenase deficiency
In summary, as this is associated with hypoxaemia, it is type A lactic acidosis.
For reference:
Equation: Pyruvate + NADH + H+ 🡨(lactate dehydrogenase)🡪 Lactate + NAD+
NAD+ is required for glycolysis to continue and produces 2ATP per mole of glucose.
Lactate is metabolised in the liver (60%) and kidney (30%)
What is a Deep Vein Thrombosis?
Deep Vein Thrombosis is a venous thromboembolism which involves the formation of a blood clot in a major deep vein. Usually in the lower limb, abdomen or pelvis.
They typically begin to form just above and below a venous valve.
Clinical features: DVTs can be asymptomatic with only 1⁄4 cases presenting with classical clinical features.
These can present with local features of venous engorgement and stasis. These include limb swelling, pain,
erythema, warmth on palpation, mild fever and tachycardia.
They can be classified as distal (i.e. in the calf – posterior tibial, anterior tibial or peroneal veins) or proximal (i.e. popliteal, femoral, profunda femoris, common femoral or iliac veins).
Aetiology is considered with Virchow’s Triad (stasis, endothelial damage and hypercoagulability).
Differentials include: cellulitis, muscle/tendon injury, mass compressing venous outflow.
Well’s Criteria:
- Active cancer (treatment/palliation within 6m)
- Bedridden recently >3d or major surgery within 4w
Calf swelling >3cm compared to the other leg
- Collateral (non-varicose) superficial veins present
- Entire leg swollen
- Localised tenderness along deep venous system
- Pitting oedema confined to symptomatic leg
- Paralysis, paresis, or recent plaster
- immobilisation of the lower extremity
- Previously documented DVT
- Alternative diagnosis to DVT as likely or more likely (-2 points)
Why is a DVT Important?
[NEED TO IMPROVE}
Can be prevented with prophylaxis & mitigation of risk factors
- It can lead to pulmonary embolism (PE) which is a major complication – approximately 10% of symptomatic patients die within 1 hour
- Amongst patients diagnosed with PE mortality is approx. 10% at 2 weeks and approx. 20% at 1 year
- DVT is a major cause of preventable complication and death after surgery
- It can lead to common long-term complications (e.g. post-thrombotic syndrome – pain, swelling, discolouration and pigmentation, varicose veins and rarely venous ulceration) + Ischaemia and necrosis.
What are some of the risk factors for hypercoagulability?
Inherited:
• Factor V Leiden (most common), Protein C and S deficiencies, antithrombin deficiency, prothrombin
gene mutation
o Factor V Leiden- clotting factor is no longer broken down by activated Protein C
o Protein C and S are Vit K dependent and cleave and neutralise V and VIII
o Antithrombin is a cofactor of heparin and inhibits thrombin
Acquired:
• Malignancy, oestrogen (HRT, OCP), dehydration, sepsis, polycythaemia, smoking, antiphospholipid
syndrome, recent trauma or surgery, obesity, pregnancy, immobility, heparin induced thrombocytopaenia, polycythaemia vera, cancer therapy (tamoxifen, thalidomide, lenalidomide), heart failure, IBD, nephrotic syndrome
How can the risks of DVT be reduced in a surgical patient?
Pre-operative:
Optimise RFs
Cease OCP 6w prior
Correct thrombophilia deficiency
Delaying surgery until resolution of hypercoagulability (pregnancy, malignancy)
Lifestyle modifications – weight loss, exercise, smoking cessation
Perioperative:
Intermittent calf compression (inflatable cushions wrapped around calves)
Post-operative:
Mechanical:
Early and frequent mobilisation
Elevation of the legs to increase venous return
Thromboembolic Deterrent Stockings (TEDs) to increase venous return
Intermittent pneumatic compression
Foot impulse device
Anticoagulation prophylaxis:
Indicated based on initialised VTE risk assessment
Low Risk 🡪 Nil
Ambulatory, Nil RF, <30m surgery
Moderate 🡪 Consider
High risk 🡪 Enoxaparin 40mg D SC 5-10days
Multiple VTE RF, major surgery (trauma, joints, abdominal, >45m)
What is Virchow’s Triad?
Definition: Virchow’s triad are the three factors that are believed to contribute to thrombosis, these are hypercoagulable sate, blood stasis (alterations in blood flow), endothelial injury.
Hypercoagulability Hereditary Inherited thrombophilia Factor V Leiden mutation (most common) Prothrombin gene mutation Antiphospholipid antibodies Protein C and S deficiency
Acquired: Age Pregnancy OCP/HRT Malignancy Obesity Surgery/trauma (especially of lower extremity) Previous VTE or arterial thrombosis Immobilisation Smoking
Stasis AF Immobility or paralysis Venous insufficiency or varicose veins Venous obstruction from tumour, obesity or pregnancy Left ventricular dysfunction
Endothelial injury Trauma or surgery Atherosclerosis (smoking, obesity, poor cholesterol) HTN Venepuncture Prior DVT
You are the general surgery resident on call. You are asked to review a 70-year-old man who is 72 hours post elective, uncomplicated colonic surgery. He has developed an acute confusion having been well and behaving normally since surgery. What are you going to do?
Phone Call:
• In what way is the patient acutely confused?
• Is there depressed level of consciousness? What is the GCS?
• What are the vital signs (including glucose)?
• Is there an obvious reason for the patient’s behaviour?
• Is the patient aggressive?
• What has been tried so far?
• Are staff or patients at risk or actually injured?
• What staff are there to help now?
I would go to review the patient, while I am walking to their bedside, I would consider the important differentials which include:
- Delirium – acute, fluctuating impairment with inattention
- Dementia – chronic, progression impairment with intact consciousness
- Psychosis – thought content/ perception issues, not necessarily confused
- Receptive or expressive dysphasia – difficulties comprehending or verbalising responses to questions
Initial Management
• If unresponsive start with ABCDE and call for senior help early
• If aggressive and risk of harm call security – 1st priority is safety
• Use de-escalation strategies: verbal, physical (if necessary), medical tranquilisation (last resort)
• Determine if the patient is delirious with the use of the Confusion Assessment Method (CAM)
• Ask the patient if they are in pain and treat appropriately
• Consider the causes of delirium, starting with major treats to life:
o Hypoxia
o Hypotension
o Sepsis
o Intracranial mass lesion
o Seizures
History - take a history from the patient and collect a collateral history from relative and ward staff.
• Review - the patient notes, observations & medication chart
• Physical Examination - perform a selective physical examination:
• Physical Examination - perform a selective physical examination:
o Vitals and GCS – also perform Abbreviated Mental Test Score (AMTS) or Quick Confusion Scale
o HEENT – nuchal rigidity, photophobia, pupil size and symmetry, fundoscopy, otoscopy
o Neuro – focal neurological deficit (esp. cranial nerve or motor deficit)
o CVS – JVP, S3, pitting oedema, new murmur
o Resp – cyanosis, respiratory distress, wheeze, crackles
o Abdo – NG tube output, costovertebral angle tenderness, guarding, rebound tenderness, jaundice, hepatomegaly, asterixis
o GU – Urine output from catheter
• Investigation - initial investigations:
o FBC, UEC, BSL, CMP, LFT
o Blood cultures (if sepsis considered)
o Urinalysis and urine MCS
o ECG & CXR
o Consider ABG (if respiratory cause considered
• If no cause found, consider further investigations
• Management
Treat any possible underlying cause identified:
Non-Pharmacological
This is the main treatment
Aim to re-orientate the patient
Close observation by a nurse in a quiet, well lit room
Have a set routine and use regular staff
Have clocks visible and give the patient their glasses or hearing aids
Have family present
Pharmacological Try to avoid using sedatives e.g. benzodiazepines As a last resort: Haloperidol 0.5mg PO or IM, single dose Risperidone 0.5mg PO, single dose Olanzapine 2.5mg PO or IM, single dose
You are the general surgery resident on call. You are asked to review a 70 year old man who is 72 hours post elective, uncomplicated colonic surgery. He has developed a fever of 38.2 degrees C, having had no fever recorded since surgery. What are you going to do?
Ask caller to recheck obs, then head over to the patient. While you are walking to the bed, consider differentials:
DDx:
Wind (Day 1-2)
Atelectasis (ventilator use, poor deep breathing)
Pneumonia (ventilator use)
Water (Day 3-5)
UTIs (catheter, urinary retention)
Wound (Day 5-7)
SSI, cannula infection
Walking (Day 5+)
DVT/PE
Wonder drugs (Anytime)
Anaesthesia
Transfusion productions
Anti-inflammatory agents
Sepsis (Anytime)
Infected foreign body (prothesis)
Leaking anastomosis
Resuscitation:
Vitals, hydration status
ABCDE
Inform surgical registrar
Hx: Surgical Hx (pre/peri/post) Onset, progression, associated features RF for infection – SSI, cannula, prosthesis, catheter, intubation, immunosuppression Medications, allergies
O/E: Possible infectious sites: Wound, cannula, catheter, prosthesis Looking for inflammation, pus, fluctuant mass, cellulitis etc. Cardiorespiratory
Ix: Initial Pathology: FBC, CRP, blood culture, U/A + MCS, wound culture CXR ECG Specific Dependent on cause: D-dimer, troponins, CT, joint aspiration (gout), LP (meningitis)
Mx: Supportive IV fluids, analgesia, antipyretic Cease unnecessary medications Remove unnecessary attachments Treat cause Infection – Abx, drain abscess DVT/PE – anticoagulation Atelectasis – incentive spirometry, deep breathing (improves over time)
You are the general surgery resident on call. You are asked to review a 70 year old man who is 12 hours post elective, uncomplicated colonic surgery. He has developed an acute episode of hypotension, which has persisted for 10 minutes. What are you going to do?
DDx: Hypovolaemia (↓ Preload) Haemorrhage Inadequate fluid replacement Cardiogenic (intrinsic pump failure) MI, CHF exacerbation, arrhythmia, cardiomyopathy, valvular disease
Distributive (↓ Afterload)
Sepsis (increased capillary permeability)
Anaesthesia related (inhibits sympathetic system causing bradycardia and hypotension)
Anaphylaxis
Obstructive
PE
Tension pneumothorax
Resuscitation:
Vitals, hydration status
ABCDE
Inform surgical registrar
Hx: Associated Sx Review: Operation report Fluid Balance Medications (diuretics, antihypertensives, analgesics) PMHx (IHD, arrhythmias, DVT/PE)
O/E:
Vitals
SIRS, dehydration, poor perfusion
Evidence of infection (inflammation, pus, fluctuant mass, cellulitis etc.)
Cardiorespiratory
Abdominal (fluid, tenderness, peritonitis)
Ix: Initial: FAST scan (free fluid) Group and save +/- cross match Septic screen – FBC, CRP, cultures, lactate CXR ECG Mx: Initial IV Fluids +/- electrolytes ? HDU referral Treat cause Haemorrhage – OT Cardiogenic – cardioversion, anticoagulation etc. Infectious – Abx PE – Anticoagulation Tension pneumothorax – chest decompression with needle thoracostomy
You are the general surgery resident on call. You are asked to review a 70 year old man who is 24 hours post elective, uncomplicated colonic surgery. His urine output has dropped from an average of 60mls per hour (>1/2ml per kg) in the first 24 hours to 20mls per hour for the last 2. What are you going to do?
Note: Normal UO is >1-2ml/kg per hr
DDx: Pre-renal Hypovolaemia (haemorrhage) Hypotension (sepsis) Low CO (HF) Renal artery stenosis (atherosclerosis)
Renal Hypoperfusion -> ischemia Glomerular disease Nephrotoxins (NSAIDs, contrast, aminoglycosides) Metabolic derangements (hypercalcaemia) Post-renal Obstruction (stones, pelvic mass) Urinary retention (neurological inhibition, overdistension, constipation)
Resuscitation:
Vitals, hydration status
ABCDE
Inform surgical registrar
Hx:
Sx
Fever, pain, confusion, dysuria etc.
AKI – weight gain, fatigue, anorexia, nausea, confusion, seizure
Hypovolaemia – SOB, weakness, light-headedness, syncope
Review
Operation report (bleeds, complications)
Fluid Balance
Medications (diuretics, antihypertensives, analgesics)
PMHx (HTN, DM, CHF, PVD, CKD, renal stones etc.)
O/E: Catheter Blockage, Kink Urine appearance Vitals Evidence of shock Abdominal Palpable bladder (obstruction, retention) Ballotable kidney Tenderness or peritonitis Cardiorespiratory
Ix: Initial FBC, UEC, eGFR, ABG, Group and Save Urinalysis Bladder scan Specific CTKUB FAST scan ECG
Mx: Initial IV Fluid challenge 500ml over 10-30m Assess for improvement in BP, UO and mal-perfusion symptoms Stop nephrotoxic medication Treat cause Hypovolaemia – fluids Urinary retention – catheterise Consider HDU if deteriorating
What are the symptoms and signs of a PE?
Highly variable, may include:
Symptoms: SOB Pleuritic chest pain (shoulder pain if diaphragm is irritated by lower lobe PE) Cough +/- haemoptysis (if infarction) Shock (if severe) – dizziness, syncope
Signs: Tachypnoeic Tachycardic Elevated JVP Parasternal heave Loud pulmonic component of 2nd heart sound (P2) Crackles (pleural effusion) Shock – hypotensive (SBP <90), altered LOC, pale/mottled skin, central cyanosis
How is a PE diagnosed?
Approach is based on assessment of pre-test probability via Modified Well’s Score:
Modified Well’s Criteria
Clinical signs/symptoms of DVT (+3)
PE is #1 diagnosis OR equally likely (+3)
Heart rate >100 (+1.5)
Immobilization at least 3 days OR surgery in past 4 weeks (+1.5)
Previous, objectively diagnosed PE or DVT (+1.5)
Haemoptysis (+1)
Malignancy with treatment within 6 months or palliative (+1)
Low risk (<2)
Consider D-dimer
Negative – PE excluded
Positive – consider CTPA (CT pulmonary angiography)
PERC (PE rule-out criteria)
All negative – PE excluded as <2% chance
PERC (+1 per criteria) Age >50 HR >100 SaO2 on RA <95% Unilateral leg swelling Haemoptysis Surgery or trauma <4wks Prior PE/DVT Hormone use
Moderate risk (2-6)
High sensitivity D-dimer
Negative – PE excluded
Positive – CTPA
High risk (>6) CTPA (D-dimer not recommended)
If patient hemodynamically unstable despite resuscitation:
Consider bedside lower extremity compression U/S to diagnose DVT to allow initiation of therapy
Refer to PERT (PE response team) who will consider risk/benefit of initating therapy
Findings:
CTPA
Embolism appear dark against contrast in pulmonary vessels
RV enlargement
V/Q scan
2nd line choice if eGF <30 i.e. renal failure or allergy to contrast
Inhale helium and inject contrast to compare V and Q
Other Ix: Pathology FBC, ESR, CRP, Troponin Elevated BNP ABG Respiratory alkalosis (hyperventilation) If massive with severe shock can be metabolic acidosis CXR Usually normal, may exclude ddx Non-specific atelectasis and effusion ECG Sinus Tachycardia (most common finding) S1Q3T3 pattern (non-specific ST segment and T wave changes) T wave inversions in anterior and inferior leads RBBB Right axis deviation
How is a PE treated?
Supportive:
A – Intubation if in CV collapse/arrest
B – High Flow O2
C – IV fluids, vasopressors (adrenaline/dobutamine), analgesia
Specific:
Severe/life-threatening:
Thrombolysis
Indicated if sustained hemodynamically instable patients
Absolute CI is bleeding, recent stroke
Alteplase 10mg IV bolus followed by 90mg IV infusion over 2h
If bleeds – FFP and anti-fibrinolytics
Embolectomy
Catheter insertion via femoral/brachial) to reach clot
Anticoagulation
Initial:
Enoxaparin (LMWH) 1.5mg/kg SC OD OR 1mg/kg BD
Ongoing: (3-6m or more)
Rivaroxaban (NOAC) – start immediately after diagnosis
OR
Warfarin – continuing LMWH for 5 days until in therapeutic range (INR 2-3)
Venous filter
Placed in inferior vena cava for 3-6months in patient with absolute CI to anticoagulation
What is a pulmonary embolism?
Definition: Obstruction of the pulmonary artery or one of its branches by material that originated elsewhere in the body.
Origin: 80% Thrombus from lower limb Proximal DVT embolises to PE in 50% Distal DVT embolises to PE in 5% Other: tumour tissue, fat, air, septic (endocarditis of tricuspid or pulmonary valves),
Classification:
Massive: acute PE with obstructive shock or SBP <90mmHg
Sub-massive: acute PE without systemic hypotension but with either RV dysfunction or myocardial necrosis
Low risk: none of the above features
Pathogenesis
As per DVT, Virchow’s triad of hypercoagulable state + stasis of blood + endothelial injury
Thus, similar RF
Pathophysiology:
PE are typically multiple, with lower lobes being involved in majority of cases
Effects are proportional to the rapidity and degree of obstruction
Physiological responses:
Due to obstruction, there is Increased pulmonary vascular resistance (PVR) which impedes RV outflow. This causes RV dilation as it attempts to maintain adequate pulmonary flow. As it fails to compensate hypotension ensues.
Obstruction results in increased alveolar dead space, i.e. ventilation with no perfusion. This results in the V/Q mismatch that triggers pulmonary vasoconstriction to optimise gas exchange. This results in hypoxaemia, stimulating respiratory drive resulting in hypocapnia and respiratory alkalosis.
In 10% of cases, small thrombi lodge distally in segmental vessels resulting in pulmonary infarction, that presents as pleuritic chest pain and haemoptysis.
What are the factors that increase the risk of formation of a Deep Vein Thrombosis?
Relevant factors on:
Modified Well’s Criteria
Immobilization at least 3 days OR surgery in past 4 weeks (+1.5)
Previous, objectively diagnosed PE or DVT (+1.5)
Malignancy with treatment within 6 months or palliative (+1)
PERC (+1 per criteria) Age >50 Surgery or trauma <4wks Prior PE/DVT Hormone use
What are the important contra-indications for a patient to have an epidural catheter placed for analgesia?
Absolute CI:
Hypovolaemia (as it may precipitate hypovolaemic shock)
Raised ICP (early morning vomiting and headache, papilledema, altered LOC, dilated unresponsive pupil)
Infection at site
Allergy to anaesthetic
Coagulopathy
- INR >1.4
- Platelet count <100
- LMWH within the last 12hrs (prophylactic dose) or 24hrs (therapeutic dose)
- Clopidogrel (within the last 7 days)
Patient refusal
Relative CI: INR 1.2-1.4 Sepsis Abnormal anatomy of spine Severe aortic or mitral stenosis
What are the common or important complications and side effects of an epidural anaesthetic?
Catheter Related
1. Epidural abscess – Back pain, infection, possible neurological deficit, if so, then MRI.
Mx: Incision + drainage + fluclox
- Infection – sore, exudate, redness, swelling, fevers.
Mx: Fluclox - Epidural haematoma – back pain, tenderness, possible motor block, of so, then MRI. Can lead to compression + ischaemia of spinal cord, leading to paralysis.
Prevention: Know coag status of pts, check site prior to administraion of anticoagulants.
Management: Notify anaesthetists, CT/MRI + surgical evacuation. - Inadequate block/no block – can feel ice cold or pain sensation.
Mx: Retry - Dural puncture – postural occipito-frontal headache, neck stiffness, photophobia, tinnitus, hearing disturbance.
Mx: Supine position, hydration, analgesia - Total spinal – ascending motor loss with LOC
Drug Effects
1. Local anaesthesia toxicity
Initial: Light headedness, drowsiness, tinnitus, circumoral tingling, metallic taste.
Undetected: convulsions, hypoventilation, arrhythmias, hypotension, tachycardia, arrest
Prevention: Aspirate catheter to check for blood, give block slowly.
Mx: Stop infusion, MET call, IV fluids, O2, airways and ventilation support, bag + mask PRN, anticonvulsants.
- High block (Unintentional anaesthetic into CSF) – Respiratory depression, weakness, bradycardia, hypotension, arm neuropathy.
Mx: Stop infusion, MET call, IV fluids, O2, airways and ventilation support, bag + mask PRN, anticonvulsants, adrenaline. - Sympathetic block: Hypotension, bradycardia, vasodilation.
Prevention: position in L lateral, administer slowly.
Mx: Stop infusion, IV fluid bolus, elevate legs, adrenaline. - Opioid effects: sedation, respiratory depression, vasodilation.
- Sub-arachnoid opioid: N/V, severe itch, drowsiness -> coma, bradypnoea -> apnoea.
Mx: Stop infusion, MET call, IV naloxone, IV fluids, O2, airways and ventilation support, ICU. - Urinary retention anaphylaxis: Confusion, pain, full bladder.
Mx: Catheterisation
What is the treatment of hypotension that is occurring in a patient who has an epidural anaesthetic in place? The patient is 12 hours post laparotomy at which a left hemicolectomy was done?
Consider causes of hypotension:
• Hypovolaemia- haemorrhage, insufficient fluid replacement/maintenance
• Cardiogenic: MI, arrhythmia
• Obstructive: saddle PE, tension pneumothorax, cardiac tamponade
• Distributive- sepsis, side effect of epidural, anaphylaxis
ABCDE assessment;
• Trial of fluid challenge 500mL N Saline
• Anaesthetics consult- consider turning down the epidural or stopping
Once stable:
• Review patients fluid balance and medications
• Treat any underlying cause as well as fluid balance
• Elevate legs
• Administer oxygen
• If refractory- get ICU involved to administer vasopressors
Jessica Redmond 29
NB:
A transient drop in blood pressure is to be expected as the sympathetic blockage produces vasodilation.
This is usually compensated for by an increase in cardiac output. If the sympathetic blockage should reach
T4 level, where the sympathetic chain to the heart leaves the spinal column the compensatory mechanism
is blocked preventing increase in heart rate. This can result in profound hypotension, bradycardia, thready
pulse or anxiety. A common problem as Epidurals tend to unmask the patients volume status.
What, chemically, is in a 1L bag of ‘Normal saline’? What is the average daily sodium requirement for a 70kg man?
Normal Saline: 0.9% NaCl 1L H2O Na 154 mmol Cl 154 mmol
Daily requirements:
2.5L of H2O (30ml/kg/day)
Na 1-2mmol/kg
K 0.5-1mmol/kg
Daily requirements for 70kg man:
2.1L (approx.2.5L) of H2O
70-140mmol/kg/day Na
70mmol K
What is the adult daily sodium requirement? What is the daily potassium requirement?
The average daily intake of sodium is 1-2mmol/kg/day. The average daily intake of potassium is 0.5-
1mmol/kg/day.
What is the adult daily potassium requirement? How do you usually prescribe this for IV infusion?
The average daily intake of potassium is 0.5-1mmol/kg/day.
Potassium is prescribed as additive to other fluids. It can be given with 5% dextrose and 0.9% saline
30 mmol KCl additive to IL N Saline or 10mmol in 100 mL
What is the adult water volume requirement per day? How would you prescribe this in IV fluid orders to someone who is nil by mouth?
The adult water volume requirement is 25-30mL/kg/day
That is a maintenance of of 1500-2500mL/day
Please write out a fluid prescripiton for 24 hours “maintenance” fluids for a patient who weighs 80kg, has no unusual insensible or measurable losses.
Note: maintenance glucose = 50-100mg/day 1% =10mg/mL
What, chemically, is in a 1L bag of “Hartmanns” solution?
Hartmann’s: 1L H20 Na 131mmol/L K 5mmol/L Cl 111mmol/L Ca2+ 2mmol/L Lactate (HCO3-) 29mmol/L (as lactate gets first pass metabolised into bicarbonate)
What, chemically, is in a 1L bag of 1/5 Saline, 4% dextrose?
4% and 1/5th NS: 1L H20 Dextrose 40g Na 30mmol/L Cl 30mmol/L
5% dextrose:
1L H20
Dextrose 50g
What are the common or important immunological complications of a blood transfusion?
Immunological complications:
• Febrile non-haemolytic transfusion reaction
• Acute Haemolytic transfusion reaction
• Urticarial reactions
• Anaphylactic transfusion reactions
• Transfusion related Acute Lung Injury
• Post transfusion purpura
• Transfusion associated graft vs host disease
• Delayed haemolytic transfusion reaction
Febrile Non-haemolytic transfusion reaction
Caused by release of cytokines by WBCs in a product that has not been leukoreduced.
Presentation: within 1-6 hours
Characterised by fever, usually with chills in the absence of systemic symptoms.
Diagnosis of exclusion
Mx is symptomatic e.g. paracetamol for fever, cease transfusion, antihistamines
Prevention: leucodepletion of blood products
Acute Haemolytic Reactions
Life threatening reaction within first 24 hours caused by intravascular haemolysis of transfused RBCs when major ABO incompatibility.
Can lead to acute renal failure, DIC and haemodynamic collapse.
Presentation: fever, chills, flank pain, oozing from IV sites, chest pain, pink serum or urine.
Classic triad = fever, flank pain and red urine.
Treatment:
Immediate communication to transfusion service
Aggressive hydration and diuresis- Normal saline
Urticarial Transfusion Reaction
Urticaria but no other allergic findings. Antigen-antibody reaction- antigens in plasma interact with recipient’s pre-existing IgE antibodies. Mast cells and basophils then
release histamine
Presentation = wheal and itch. No wheezing, angioedema, hypotension
Mx:
Can continue remainder of blood product if no wheezing, angioedema, hypotension
after antihistamine.
Anaphylactic Transfusion Reaction Various mechanisms: • Recipients with IgA deficiency and IgG anti-IgA antibodies • Anhaptoglobinaemia • Passive transfer of food allergen
Presentation = rapid onset. Angioedema, wheezing, hypotension. May occur in IgA deficient individuals.
Management: cease transfusion, adrenaline repeat every 5 minutes as necessary IM, antihistamines, vasopressors, airway management.
TRALI
Life threatening occurs when recipient neutrophils are activated by the transfused products in short time or have underlying cardiovascular disease. Neutrophils release inflammatory mediators and damage pulmonary capillaries.
Presentation: fever, chills, respiratory distress, hypotension.
Mx: supportive, may include intubation and mechanical ventilation.
Post-transfusion Purpura
Occurs after transfusion of any platelet containing produce. Caused by platelet-specific alloatbodies.
Presentation–> 5-10 days post infusion
-bleeding
-thrombocytoapenia
Mx: high dose IVIg for several days
Transfusion Associated Graft vs Host Disease
Donor lymphocytes recognise recipient cells as foreign, initiating GvHD-
(usually, donor lymphocytes destroyed by recipients immune system)
Risk factors:
-immunocompromised pts
-specific type of partial HLA matching between donor and recipient
Presentation- 4- 30 days post op
• Fever
• Erythematous, maculopapular rash –> TENS
• n/v
• Abdo pain often RUQ
• Produse diarrhoea
• Cough
• Ix: pancytopaenia- hypocellular marow, deranged LFT, electrolyte abnormalities
Dx is supported by skin biopsy and HLA phenoytping
Mx- no effective treatment- almost always universally fatal
Prevent by irradiating products to inactivate T cells
Delayed Haemolytic Transfusion Reaction
Haemolytic tractions that occur more than 24 hours after completing the transfusion.
Often, they occur days to weeks later. They are typically gradual and less severe and
often clinically silent.
Autoantibodies to minor antigens e.g. Rh, Kell ,Duffy, Kidd.
Presentation: 2-30 days post transfusion.
-falling Hct, anaemia
Fever- mild
Unconjugated bilirubin increase
Mx:
No treatment required in the absence of rapid haemolysis
Future avoidance of implicated antigen (must report to patient)
What are the common or important non-immunological complications of a blood transfusion?
Non-immunological complications: • Infection/Sepsis • Transfusion related circulatory overload (TACO) • Primary hypotensive reactions • Iron overload
Infection/Sepsis
Caused by transfusion of a product that contains a microorganism.
Presentation: fever, chills, hypotension
Broad spectrum antibiotics and haemodynamic support.
TACO
Pulmonary oedema due to volume excess or circulatory overload.
Diuresis, supplementary oxygen.
Presentation: respiratory distress, rales, SOB, orthopnoea, PND, hypertension, Abnormal CXR, hypoxaemia.
Prevention: non rapid infusion rates
Drop in blood pressure without other causes of hypotension. Exclude TRALI, sepsis, anaphylaxis.
Most often reported in platelet transfusions.
Iron Overload
In chronically transfused patients- e.g. sickle cell, thalassemia, myelodysplastic syndrome.
What interventional (non-pharmaceutical) options are there for treating chronic (not acute) coronary artery insufficiency?
Definition: Occurs due to insufficient blood flow through one or more coronary arteries usually due to atherosclerotic disease.
Classification: Stable angina Retrosternal chest discomfort Exacerbated by exercise Relieved by rest or GTN May be silent in diabetics, elderly and females Acute Coronary Syndrome Unstable angina STEMI NSTEMI
Indications:
Patient with chronic stable angina in which:
Failure of medical therapy
Maximum does not improve symptoms
Maximum intolerable
Significant risk factors on non-invasive testing:
Smoking
Obesity
Dyslipidaemia
Co-morbid DM, HTN
ST changes on stress ECG
Patient categories with better survival post-surgical intervention
i.e.
Stenosis of Left Main
Triple vessel disease (RCA, LAD and circumflex)
Two vessel disease (proximal LAD and another)
Method:
Percutaneous coronary intervention (PCI)
Indications:
1-2 vessel disease
Suitable anatomically
Low risk CVD
CI:
Anatomically unsuitable (long, near bifurcations)
Procedure:
Insertion via femoral artery
Coronary guidewire inserted into stenotic area and balloon catheter is pushed above.
Inflation results in plaque compression and stretches stenosis to allow stent placement
Requires dual antiplatelet therapy + heparin post-op
Cx:
Thromboembolism – MI, stroke
Access site – haematoma, pseudoaneurysm, perforation
Contrast induced AKI
Restenosis (1/3 in 6 months)
Stent thrombosis
Advantages:
Minimally invasive (better for comorbidities i.e. pulmonary disease)
Cheaper
Faster recovery time
Less adverse events
Superior for acute STEMI
Disadvantages:
High re-stenosis rates
Not suitable for 3 vessel disease or LMCA
Not suitable for non-compliant patients i.e. anti-platelets
Potentially incomplete revascularisation
Coronary Artery Bypass Graft
Indications:
Failure of medical therapy
Three vessel disease or two vessel involving proximal LAD
Failed PCI or restenosis with angina
High risk CVD
CI:
Unfit for surgery
Procedure:
Internal vein used to bypass stenotic artery
Most commonly long saphenous vein, radial artery, mammary artery
Must continue strict lifestyle modification/therapy
Cx:
Thromboembolic events – MI, stroke
Haemorrhage
Pericarditis
Graft failure
Advantages:
More complete revascularisation
More effective for multi-vessels
Better long-term results – lower risk of recurrent angina
Mortality benefit in diabetic patients
Disadvantages:
More peri-operative complications – death, stroke, GA, surgical risks
Longer recovery and hospital stay
What are the indications for a pneumonectomy in a patient who has been found to have a primary non-small cell lung cancer?
Indications: Extent Stages: 1-2 considered operable 3A limited patients operable 3-4 inoperable
Patient preference
Fitness
Co-morbidities that CI surgical Mx – cardiac disease, CCF, end-stage renal disease
Recovery and rehabilitation
Psychosocial impact
Predicted post-operative FEV1 >80%
Requires adequate respiratory reserve to tolerate pneumonectomy and maintain quality of life
CI:
CI in patients with disease extending past the diaphragm:
Intra-abdominal
Contralateral hemithorax
Invading structures of the mediastinum – aorta, IVC, heart, oesophagus
Ribs
Malignant disease
Surgical options:
Wide resection + mediastinal lymph node sampling
Segmentectomy
Lobectomy
Sleeve resection
Involves removing infected lobe and primary bronchus and then the remaining lobes are re-joined to the remainder of the primary bronchus.
Lung cancer: Non-small cell cancer (NSCLC) 85% Adenocarcinoma 40% Squamous cell carcinoma 25% Large cell (undifferentiated) 10% Small cell lung cancer (SCLC) 10%
What are the principles of a heart lung bypass perfusion circuit (machine)?
Function:
Temporarily replaces the function of the heart and lungs during surgery to maintain systemic circulation and oxygenation.
Method:
Removes blood from the intravascular space, commonly RA or SVC + IVC.
This is filtered through a membrane oxygenator which both oxygenates blood and removes CO2.
It is then passed through a heat exchanger and then a filter to remove air bubbles.
The blood then returns to the body via the ascending aorta, which is pumped under pressure.
Method: Pre-operative Anticoagulation – high dose Heparin CPB circuit primed with crystalloid, heparin +/- mannitol Systolic BP is lowered Prior to cannulation SBP 80-100 During bypass MAP 50-70 Cardioplegic solution administered (K+/Mg2+, procain) – decrease myocardial oxygen demand
During
Lungs are deflated by turning off ventilator
BP maintained with vasodepressors and dilators
Induce slight hypothermia (28-34) – prevent ischemia
Post-operative
Air excluded from cardiac chambers and aortic clamp removed
May require cardioversion or pacing if arrhythmia or heart block
Machine gradually switched off
Cannulas removed and anticoagulation reversed with protamine sulphate
What are the advantages and disadvantages of using a mechanical valve to replace an aortic or mitral valve, rather than a tissue valve?
Mechanical
Advantages: Durability Sterile – decreased risk of infection Availability Simple procedure
Disadvantages: Thrombogenic – increased VTE risk Lifelong Warfarin Bleeding risk Sometimes audible when open/close Endocarditis
Tissue
Advantages:
Reduced VTE risk
Reduced anticoagulation need – decreased bleeding risk
Disadvantages: 10-15-year lifespan May need replaced – mortality risk Higher rates of infection Difficult procedure
Therefore, mechanical valves are normally used in younger patients and tissue valves in older patients.
What are the indications for surgery in a patient who has been stabbed in the chest?
There are 3 urgent considerations for surgery:
- Cardiac Tamponade
a. Dx: FAST scan
b. Initial MX ED = pericardiocentesis and catheter
c. Definitive Mx – thoracotomy – release tamponade, control haemorrhage
i. Indications: pts with pulseless ECG activity - Significant haemorrhage
a. >1-1.5L initial loss or >200mL/hour ongoing for 2-4hrs
b. Suggests major vascular wall injury unlikely to stop without surgical - Massive air leak
a. One present during all respiration – suggests major bronchial injury - Flail chest
Other Considerations for Surgery
Direct injury
Ruptured blood vessel – endovascular stent or open repair
Aorta – if unstable BP emergency surgical repair
Trachea/bronchus – tube thoracotomy then open repair
Diaphragm – laparoscopic repair
Oesophagus – open repair
Pulmonary laceration – open repair
Haemothorax
PC: reduced chest expansion/breath sounds, dull to percussion
Mx: Emergency thoracotomy
Pneumothorax
PC: persistent air leak from chest tube
Mx: thoracotomy
Overview of Chest Wall Injury
- Many penetrating chest wall injuries do not require surgery and can be managed conservatively with:
- Serial CXRs
- Simple chest tube thoracotomy
(15-30% of chest wall traumas will require surgery)
Approach to Penetrating Chest Injuries
- Primary survey and resuscitation
- Detect and immediately treat life-threatening conditions such as:
- Tension pneumothorax
- Massive haemothorax
- Open pneumothorax
- Cardiac tamponade
- Flail chest
Rapid diagnostic testing:
- CXR
- FAST Scan
- ABGs
You are the ED intern on duty. You are asked to see and treat a 20 yoa man who has broken his nose playing football 3 hours previously. It has not stopped bleeding. You can ignore all other issues other than those related directly or indirectly to do with his nose. How will you proceed to assess and treat his nosebleed?
Assessment:
Primary survey (ABCDE)
Basic manoeuvre: patient sits upright, bends at hips, and applies digital pressure on cartilage for >10mins.
History
PC: duration, severity, localisation (nose/throat), causative factor (trauma, coagulopathy, medications, environmental, iatrogenic)
PMHx: previous bleed and method of control
Evidence of anaemia
Examination
GI: nasal symmetry, deviation, tenderness, associated injuries
Identify source of bleeding:
Nasal speculum +/- suctioning
To reduce pain o/e consider 5% lignocaine with 0.5% phenylephrine nasal spray
Posterior bleeding:
Suggested by failure to identify anterior, bleeding from both nares, visualisation of blood in posterior pharynx
Refer to ENT for rigid endoscopy
Mx:
Pain management
Cauterisation
Apply silver nitrate stick 10-15s (reacts with mucosal lining to produce chemical burn)
Never cauterize both sides of septum to avoid perforation
Nasal packing
Anterior
- Rapid Rhino
–Nasal tampon with inflatable balloon coated in a compound that acts as a platelet aggregator, may remain in place for 3-4days
–Ribbon gauze soaked in paraffin paste or Kaltostat
Posterior (if bleeding continues despite anterior packing)
- Foley catheter
Inserted along floor of nasal cavity into the posterior pharynx then inflated to sit in nasopharyngeal space. Re-insert anterior packing.
As increased risk of hypovolaemic shock – insert IV cannula, send bloods (FBC, coagulation studies, cross-match), commence IV fluids
Surgical options (refer to ENT)
Arterial ligation (SPA – most common, external carotid, internal maxillary)
Embolization (useful if failed SPA ligation or unfit for general anaesthesia)
For reference:
Definition: bleeding from the nasal cavity and/or nasopharynx (common due to rich vascular supply)
Classification:
Anterior (90%)
Located in anteroinferior nasal septum
Formed by anastomoses of four arteries “GASS” – greater palatine, anterior ethmoidal, sphenopalatine, septal branches of superior labial
Posterior
Located in inferior lateral nasal wall, posterior to inferior turbinate
Formed by anastomoses of two arteries – Sphenopalatine and pharyngeal
Greater risk of airway compromise, aspiration and uncontrollable haemorrhage
A vs P
Division occurs at piriform aperture (pear-shaped bony inlet of the nose formed by the nasal and maxillary bones)
You are a GP. You are consulted by a 55 yoa man who has a hoarse voice. What features of his history must you establish before deciding on your next steps?
History: • Duration of voice complaints and whether the symptoms are constant or intermmittent • Characteristics of onset o Sudden or gradually progressive o Pattern- worse on voice use, worse in morning • Potential triggering factors o Vocal abuse o Concurrent URTI o Change in medications o Exposure to known allergens or toxins • Exacerbating and ameliorating factors o Improvement with voice rest o Fatigue with use • Other head and neck symptoms o Dysphagia o Otalgia o Odynophagia (pain on swallowing) o Bleeding o Throat pain o Post nasal drip • History of smoking and alcohol use • Red Flags- (when not a/w acute RTI) o SOB o Stridor o Cough o Haemoptysis o Throat pain o Dysphagia o Odynophagia o Weight loss • Use of meds affecting voice o Steroids - inhaled • History of reflux or sinonodal disease • History of past surgery involving neck, base of skull or chest o Thyroid o Carotid o Cervical spine • History of trauma and endotracheal intubation • Occupation, hobbies, habits impacting voice use • Medical comorbidities that can affect voice – rheumatoid, tremor, hypothyroidism
Following Hx I would also like:
• Physical examination
• Referral for ENT for laryngoscopy +- FNA and imaging
• Any patient with hoarseness >3 weeks needs CXR
• Indications for referral
o SCALD
o S- smoker, stridor
o C- constant/persistent hoarseness, coughing up blood
o A- acute onset (not related to URTI or EtoH)
o L- Loss (of weight)
o D- Dyspnoea or dysphagia
• Early referral and detection essential
o 70-90% 5 year survival, worse prognosis in stages 3-4
Ddx
Causes of hoarseness may be considered in the following categories:
• Acute laryngitis, which is self-limited and related to acute respiratory illness or acute voice misuse
o Moraxella catarrhalis, haemophilus influenzae, pneumococcus isolated from larynx
o Acute vocal strain – such as screaming or protracted coughing= can result in submucosal
microtrauma of the vocal fold
• Chronic laryngitis, which is related to irritants, reflux, chronic infection (such as fungal), or habitual
vocal misuse- beyond 3 weeks
o Irritants
o Laryngopharyngeal reflux
o Muscle tension dysphonia
• Benign vocal fold lesions
o Polyps and nodules
• Malignancy
• Neurologic dysfunction
o Unilateral or bilateral paralysis
o Parkinsons
o MND
o MG
o Tremor
• Non-organic (“functional”) issues
Laryngeal anatomy:
Function:
Phonation (production of a primary vocal tone at the level of the vocal folds)
Respiration
Swallowing (laryngeal elevation, posterior deflection of the epiglottis, inhibition of respiration, closure of the vocal folds to prevent aspiration of ingested material)
Gross:
Extends from the epiglottis to the inferior aspect of the cricoid cartilage
Divided into supraglottis, glottis (location of true vocal cords), subglottis
Supply:
Above vocal cords: superior laryngeal artery (branch of superior thyroid)
Below vocal cords: inferior laryngeal artery (branch of inferior thyroid)
Innervation:
Motor: recurrent laryngeal (branch of vagus), except for the cricothyroid muscle supplied by external laryngeal
Sensory:
Above vocal cords: internal laryngeal nerve (branch of superior laryngeal)
Below: recurrent laryngeal
You are the ED intern on duty. Your patient is a 30 year old man who has had a sore throat for 3 days, which is getting worse. He has difficulty opening his mouth, ear pain on the right and can’t swallow his saliva. On examination he has a temperature of 39 degrees celcius, pulse 115bpm. Oral examination reveals a unilateral, left sided tonsillar swelling with a diffuse, oedematous bulge superior and lateral to the tonsil. The uvula is deviated to the right.
What is the diagnosis?
What is the cause of the problem?
Dx: Peritonsillar abscess
DDx:
Epiglottitis: rapidly progressive, younger children
Retropharyngeal abscess: neck pain and tenderness, minimal peritonsillar findings
Parapharyngeal abscess: bulging posterior to tonsillar pillar, rather than superior
Tonsillopharyngitis: bilateral swelling, no trismus, no palpable fluctuance
Peritonsillar cellulitis: U/S, response to Abx
Clinical diagnosis confirmed by collection of pus on drainage. If diagnostic uncertainty U/S or trial of Abx may help.
Q2. Pathogenesis
Infective
Preceding tonsillitis or pharyngitis that progresses from cellulitis to phlegmon to abscess
Most common pathogens include S.Aureus, B-haemolytic strepoccoci, S.Pyogenes, H.influenza
Non-infective
Due to obstruction of the Weber glands (salivary glands in soft palate), associated with smokers
For reference: Peritonsillar abscess
Definition: Peritonsillar abscess (quinsy) is a collection of pus located between the capsule of the palatine tonsil and the pharyngeal muscles in the superior pole.
Epidemiology:
Most common deep neck infection
20-40s, M>F, smokers
Hx:
Unilateral sore throat +/- ipsilateral ear pain
Fever
Muffled “hot potato” voice
Dysphagia with pooling/drooling of saliva, odynophagia
Trismus (inability to open mouth due to inflammation and spasm of the internal pterygoid muscle)
Distinguishes PTA from severe pharyngitis or tonsillitis
O/E:
Swollen, erythematous, fluctuant tonsil (note: bilateral PTA is rare)
Deviation of the uvula to the opposite side
Cervical lymphadenopathy
Ix:
CT – fluid collection with large and inflamed tonsil
Cx:
Retropharyngeal effusion or abscess
Septic thrombophlebitis of internal jugular vein
Extension into surroundings 🡪 airway obstruction
You are the ED intern on duty. Your patient is a 30 year old man who has had a sore throat for 3 days, which is getting worse. He has difficulty opening his mouth, ear pain on the right and can’t swallow his saliva. On examination he has a temperature of 39 degrees celcius, pulse 115bpm. Oral examination reveals a unilateral, left sided tonsillar swelling with a diffuse, oedematous bulge superior and lateral to the tonsil. The uvula is deviated to the right.
What is the diagnosis?
What is the management?
Dx: Peritonsilar abscess (quinsy)
Mx:
Resuscitation (ABCDE):
Airway - ?ET intubation
Breathing – pulse oximeter, O2 as required
Circulation – IV access, bloods (FBC, UEC, culture), IV fluids
Immediate:
Analgesia
Empirical IV Abx – benzylpenicillin or clindamycin + metronidazole
Definitive:
Needle aspiration OR incision and drainage WITH MCS (outpatient or in OT)
Emergency tonsillectomy
Significant airway obstruction, recurrent episodes, failure of drainage
On discharge:
Follow-up within 7 days
Return if worsening or recurrence of symptoms
Oral Abx 10-14days
You are on-duty in A&E. Your next patient is a 3 year old child with rapidly progressive airway obstruction, looking toxic, drooling, tachycardic and tachypnoeic, sitting up leaning forward looking worried. The child is not immunised.
What is the most important differential diagnosis?
What will you do?
The most important differential diagnosis is epiglottitis.
Epiglottitis describes inflammation of the epiglottis and adjacent supraglottic structures. Without
treatment, epiglottitis can progress to life-threatening airway obstruction. It is often caused by
Haemophilus influenza type B (which is part of the immunisation program thus often only seen in
unvaccinated children).
Maintenance of the airway is the first priority for patients with epiglottitis. In patients with signs of total or
near-total airway obstruction, airway control should occur prior to diagnostic evaluation.
• Defer attempts at visualizing the epiglottis (tongue blade or any other instrument) or invasive procedures
(eg, IV placement, phlebotomy, or any other painful or frightening intervention) until after airway
assessment and management
• Prepare to manage the airway and immediately involve airway specialists (anesthesiologist or critical care
physician and otolaryngologist) whenever available
Patient not able to maintain the airway: Attempt bag-valve mask ventilation
• Unable to oxygenate (pulse oximetry lower than high 80s or falling): Attempt endotracheal intubation by
rapid sequence intubation first but be prepared to establish a surgical airway (eg, needle cricothyrotomy or
surgical cricothyrotomy)*
• Able to oxygenate (pulse oximetry high 80s and steady or improving): Endotracheal intubation by the
most capable provider, preferably in the operating with an otolaryngologist present
Patient able to maintain the airway:
• Provide supplemental humidified oxygen and maintain the child in a position of comfort with the parent
present (eg, sitting on the parent’s lap on the stretcher)
• Keep the patient in a setting where the airway can be rapidly managed if necessary with capable
personnel and specialized airway equipment constantly available
• Soft-tissue radiograph of the lateral neck (portable if possible) may be helpful but necessary personnel
and equipment to manage an acute airway event must remain with the patient at all times during the
imaging process.
• Do not image patients with severe respiratory distress in whom it will delay definitive airway
management
Radiographic findings of epiglottitis: Enlarged epiglottis (“thumb” sign), loss of vallecular air space,
thickened aryepiglottic folds, and/or distended hypopharynx
Attempts at direct visualization are only appropriate in patients with no stridor or stridor without
significant distress, no increase in symptoms with agitation, and no cyanosis
In children <6 years of age with confirmed epiglottitis or older children who are toxic-appearing or have
>50 percent obstruction of the lumen by direct visualization, perform endotracheal intubation in the
operating room with an otolaryngologist present
Under GA: examine throat + swabs, intubate, take blood cultures
ICU
Start IV Abx – ceftriaxone or cefotaxime.
Lana is a 52 year old female who is day 1 post total thyroidectomy. You are the RMO in-charge of the ward and have been called by the NUM to assess Lana urgently. Lana is having difficulty breathing and her neck looks significantly swollen. What is the most likely diagnosis? What will you do?
Post-operative cervical haematoma (usually in paratracheal area) – rare but life-threatening complication.
Management:
Call senior + transfer to OT
Resuscitation (ABCDE):
Airway
If non-critical may perform in OT, if critical perform at bedside
Establish airway:
Relieve pressure on airway by removal of sutures and evacuation of haematoma. Keep pressure without occluding trachea or carotids.
Attempt intubation
If all else fails, then cricothyroidotomy
Breathing – immediate high flow O2
Circulation - establish IV access, control haemorrhage
OT
Re-opening and inspecting for location of bleed
Place drain to remove fluid collecting in bed of haematoma
Transfer to ICU for observation
For reference: Post thyroidectomy haematoma
Description:
Haematoma is a collection of blood within the tissue
Due to major vessel ligature, re-opening of cauterized veins, bleeding from traumatized sternocleidomastoid muscle.
Causes life-threatening airway obstruction from direct tracheal compression and lymphatic/venous congestion
Presentation:
Commonly 12-24hrs post-op
Progressive neck swelling
Signs of airway obstruction
Respiratory distress (dyspnoea, stridor, anxiety, tachycardia)
Hypoxia (tachycardia, sweating, irritability, confusion)
Bleeding along suture line or significantly into drain
Transient vocal cord paralysis
Heather is a 4 year old girl who had a tonsillectomy 7 days ago. She presents to the ED with bleeding per orally. She has bled around 2 cupfuls of fresh blood in 30 minutes. You are one of the emergency department doctors. Heather is in waiting room.
What is the diagnosis?
What is your management plan?
Dx: Secondary (>24hrs) post-op haemorrhage
Occurs when fibrin clot (eschar) separates between day 5-10 exposing vessels to local trauma.
There is significant vascular supply to the tonsil (primarily from tonsillar branch of the facial artery) which predisposes it to haemorrhage.
Mx:
Admit
Resuscitation (ABCDE):
Airway – position patient forward, encourage spitting blood into bowel, suction as required
Breathing – O2, OBS
Circulation – IV access, bloods (FBC, coagulation studies, G&S), IV fluids
Immediate:
Urgent ENT review
IV analgesia
NBM
24hrs monitoring
Definitive:
Examine oropharynx for source of bleeding – visualise greyish white eschar
Apply pressure to tonsillar fossa with gauze soaked in lignocaine and adrenaline
If non-resolving 🡪 OT:
Electrocautery or silver nitrate
Topical haemostasis (topical thrombin, fibrin sealant)
Suture ligation
Surgical embolization
45 year old male comes to your GP practice with a 2 month history of hoarse voice. He has recently developed a swelling on the left side of his neck. He is a heavy smoker and consumes substantial alcohol daily. You are the doctor examining him. What is the most likely diagnosis and examination findings and what are relevant investigations/referrals for him?
Dx: Squamous cell carcinoma (SCC) of the larynx
O/E:
General observation
Hoarseness or vocal change
Respiratory distress, noisy breathing
Cachexia, weight loss
Head and neck
Ear: otalgia
Oral cavity and oropharynx: masses or leucoplakia
Laryngeal inlet: lesions of vocal cords, tumours
Neck: cervical lymph nodes or masses (size, location, shape, mobility, firmness, tenderness)
Respiratory
Signs of distress: stridor, diminished breath sounds, obstruction
Refer to ENT for Ix
Urgent laryngoscopy
Visualise lesions and document vocal cord mobility
Imaging (assess local infiltration, LN involvement, presence of Mets or secondary tumours)
CT contrast neck and chest
Whole body PET/CT scan
FNA of neck mass or cervical LN (under U/S guidance)
Rigid direct laryngoscopy with biopsy (FNA or excisional) under GA
For reference: Laryngeal cancer
Background:
Laryngeal cancer approx. 20% of all cases of head and neck cancer
90-95% are SCC
Glottic (59%) > Supraglottic (40%) > subglottic (1%)
Metastatic patterns:
Larger (especially if located in supraglottis) spread lymphatically to the neck
Lungs most common site for metastases
Prognosis: If localised 5-year survival 77%
Risk factors: M>F >40y/o 40 pack year, EtOH Chronic viral infection – HPV, HIV
Pathophysiology:
Chronic tobacco exposure irritates the epithelial surfaces causing pre-malignant changes of hyperplasia and dysplasia, pre-disposing the area to malignancy.
If alcohol is consumed as well, it more than doubles the risk compared to exposure to one factor alone.
Presentation:
Hoarseness >3wks (early in glottic, late in supraglottic and subglottic)
Dysphagia, odynophagia
Otalgia (referred ear pain via CN X – suggests advanced disease)
Cervical lymphadenopathy (poor prognosis – may be only sing of supraglottic lesion)
Stridor, dyspnoea
Haemoptysis, chronic cough
Unexplained weight loss, fatigue, night sweats
DDx: Chronic Laryngitis Polyps on the vocal cords Reinke’s oedema Contact granuloma HPV papilloma’s Benign tumours (rare)
45 year old female comes to your GP practice with a 1 year history of a midline neck swelling. She also gets tremors and says that her heart races at times and has lost some weight that she is pleased about.
What are the relevant investigations that are related to this case?
Pathology: Thyroid function tests (TFTs) Decreased TSH Increased free T4, free T3 Thyroid autoantibodies (positive) Thyroid peroxidase (TPO) antibody TSH-receptor antibody Other: FBC (normochromic normocytic anaemia, mild leukopenia) LFTs (elevated ALT and AST) Hypercalcemia
Imaging:
Radionuclide thyroid scan (Technetium-99m)
Normal or elevated uptake
Homogenous: graves
Heterogenous: toxic multinodular goitre
Decreased uptake: thyroiditis, exogenous thyroid hormone
Thyroid U/S +/- FNA
Indicated if suspicious nodules (i.e. macrocalcification, irregular border, increased vascularity)
ECG (document any arrhythmias or other changes)
For reference: Hyperthyroidism
Causes:
Autoimmune (Grave’s disease) – most common
Thyroid-stimulating antibodies (IgG) bind to TSH receptors causing excess thyroid hormone secretion. Over time the thyroid hypertrophies and becomes diffusely enlarged.
TSH-secreting pituitary adenoma
Toxic multinodular goitre
Thyroiditis
Exogenous intake of thyroid hormones
Clinical features:
Increased MR: weight loss, heat intolerance, sweating
CNS overactivity: tremor, nervousness, irritability, lethargy
Beta-adrenergic sympathetic overactivity: palpitations, tachycardia, arrhythmias
Eye signs: soft tissue oedema, exophthalmos, diplopia, ophthalmopathy
Thyroid storm (thyrotoxic crisis):
Precipitated by non-thyroid surgery, major trauma, infection, imaging with iodinated contrast
Features: insomnia, anorexia, vomiting, diarrhoea, sweating, fever, tachycardia
Liam was involved in a fight yesterday at the local pub and was slapped in the face. He had a sharp shooting ear pain following the slap and says that he cannot hear too well since then. He also says that he cannot pop his ear drums since that incident. He comes to your practice.
What is the most likely diagnosis and what will you expect from the examination point of view?
What will be your advice?
Traumatic perforation of tympanic membrane
O/E:
Anatomic inspection:
External ear: Haematoma, bruising, trauma, discharge
Otoscopy:
TM perforation (central or marginal – unsafe as allow skin invasion into middle ear)
Blood crust
Physiological assessment:
Hearing
Positive Rinne’s test (conductive: bone conduction > air)
Weber’s test (conductive: loudest on ipsilateral side to affected ear)
Facial nerve (CN VII) palsy
Symmetry, facial movements, taste
Advice:
Most perforations heal spontaneously within 8wks
Keep dry and clean to avoid infection (i.e. use plug when showering)
If signs of infection (pain/discharge) 🡪 Abx
Follow-up in 4-6wks
If not improved, refer to ENT for surgical management (tympanoplasty)
For reference: Rupture tympanic membrane
Causes:
Blunt force trauma to external ear canal (car accident, blow to side of head, explosion)
Penetrating trauma (cotton buds, iatrogenic during irrigation or FB removal)
Barotrauma (air travel, scuba diving)
Red flags:
Significant polytrauma
Battle’s sign (mastoid ecchymosis indicating skull base fracture)
Facial nerve palsy
Presentation:
Sudden severe pain (otalgia)
Serosanguinous drainage (fresh blood leakage)
Conductive hearing loss (sound does not reach the inner hear)
Tinnitus (ringing or buzzing in ears)
Fullness in the ears (aural fullness)
Disequilibrium/imbalance
You are working as rural relief resident in a remote town GP practice. A 12yo aboriginal boy is brought in by his mum to see you as he is complaining of poor hearing in his right ear that is getting gradually worse. He gives a history of constant offensive discharge from the same ear with frequent infections. You look into his ear and see some dry white crusting wax at the top of his ear drum.
Cholesteatoma (abnormal growth of epithelium in the middle ear that results in bone erosion and infections)
Actions:
Initial workup:
Otoscopy: (confirm diagnosis)
Attic (posterosuperior portion of TM) retraction containing keratin
Migration of keratin through perforated TM
White crust within or behind intact TM
Audiometry (assess hearing loss)
CT temporal bone (assess for extent and complications)
Ear swab of discharge
Mx: Immediate: Analgesia (paracetamol/NSAID) Topical Abx +/- wick Definitive: Refer to ENT Mastoidectomy Canal wall up (closed) – heals better, but risk that disease will recur Canal wall down (open) – decreases risk of recurrence, will need to wear earplug when swimming and have wax cleaned Tympanoplasty
For reference: Cholesteatoma
Causes:
Eustachian tube dysfunction (from allergies, sinusitis, chronic middle ear infections)
Perforated TM due to infection of injury
Congenital remnant of skin trapped in middle ear
Implantation of keratinocytes following otologic surgery or traumatic blast injury
Presentation:
Hearing loss or tinnitus
Chronic offensive discharge
Otalgia, vertigo, facial nerve palsy
You are working in emergency and you see a 4yr old boy with fever, left otalgia and acute left facial nerve palsy.
a) What is the most likely diagnosis?
b) What condition could this develop into that causes swelling behind the ear and the ear to stick out/forwards?
VIVA II
a) What is the most likely diagnosis?
b) How would you manage this boy?
VIVA III
a) What is the most likely diagnosis?
b) What dangerous complications can result from this condition?
VIVA I
Dx: Acute Otitis media
The condition that this could develop into that causes swelling behind the ear and the ear to stick out/forwards is Mastoiditis – a bacterial infection of the mastoid cells surrounding the inner and middle ear, causing swelling behind the ear and the ear to stick out or forwards.
There will be oedema, erythema and tenderness over the mastoid process.
This is also managed with analgesia and antibiotics.
VIVA II
Dx: Acute Otitis media
Management:
- Acute pain relief with analgesia (usually paracetamol +/- ibuprofen)
- Antibiotics (usually penicillin/amoxycillin. Refer to local guidelines).
- As the patient will likely have an ongoing viral respiratory infection, the patient should also be encouraged to drink plenty of fluids and rest.
- ENT referral for surgical mx
VIVA III
Dx: Acute Otitis media
Dangerous Complications: Intratemporal complications that can result include: 1. Hearing loss 2. Balance and motor problems 3. Tympanic membrane perforation 4. Cholesteatoma 5. Mastoiditis 6. Seventh cranial nerve palsy
Other complications include:
- Meningitis
- Epidural abscess
- Subdural abscess
- Brain abscess
- Lateral sinus thrombosis
- Cavernous sinus thrombosis
- Carotid artery thrombosis
For reference: Acute otitis media
Clinical features: Ear: 1. Otalgia (ear pain) 2. Postauricular tenderness 3. Decreased hearing
- Physical examination will reveal a bulging, opacified tympanic membrane with decreased mobility.
- The membrane may be white, yellow, pink or red.
- Diagnosis is generally made with conventional otoscopy
Resp:
1. Usually ongoing viral respiratory infection
Neuro:
1. Facial nerve palsy
Systemic:
- Irritability/Lethargy
- Fever
GI:
- Anorexia
- Nausea/Vomiting
Protruding auricle/external auditory canal oedema
Ix (not routine):
CT -> bony destruction of mastoid air cells
Pathophysiology: Preceding event (e.g. URTI) -> inflammatory response. Inflammatory oedema obstructs the isthmus in the eustachian tube -> accumulation of secretions in middle ear space -> viral/bacterial growth.
Pathogens:
Viral (most common): RSV, Rhinovirus, enterovirus
Bacterial: S.pneumoniae (50%), H.influenzae (45%), Moraxella, GAS
You are the surgical ward resident. A nurse calls you saying that a patient who had frontal sinus
surgery this morning had a fall in the toilet. They have also been bleeding intermittently throughout the
day. What will you do?
While I am still on the call, I would ask the nurse to check for any surrounding dangers, rechek the patient’s observations, and ensure their immediate stability.
I would urgently review this patient as there is a concern this patient may be hypovolaemic due to post-operative blood loss.
Whilst on my way to the patient's bedside, I will be considering differentials categorised by system: Cardiovascular System -Hypovolaemia - MI - Arrhythmia - Fluid imbalance or acid/base imbalance - Exacerbation of valvular pathology
Neurologic System
- Vasovagal or micturition syncope
- CVA
- Delerium
- Seizure
- Also consider psychiatric aetiology
Respiratory System
- PE
Musculoskeletal System
- Muscle weakness
- Functional issue e.g. slippery surface/severe pain
Endocrine
- Hypoglycaemia
Once at the patient’s bedside, or rather toiletside, I would begin by ensuring their:
1. Airway is patent and assess for signs of bleeding
2. Breathing and Circulation are adequate
Look for chest rise, whilst listening and feeling for breath sounds with my ear and assess fluid status by checking pulse, cap refill and checking urine output.
3. Checking for disability. GCS (pupils, limb strength, motor/sensory assessment, focal deficit assessment)
- Checking exposure
Examine for injuries sustained during fall (head/neck, fracture, bruises/lacerations)
Secondary survey: Hx - Mechanism of fall - Precipitants/preceding symptoms (straining, constipation, standing, dizziness, palpitations, SOB) - LOC, time to recovery - Witnesses? - Subsequent pain/concerns
O/E
- Cardiorespiratory
- Neurological + cranial nerve
- Surgical site? bleeding
Review:
- OBS
- Operation report
- Progress notes
- Fluid balance
- Medications (particularly anti-hypertensives)
- Consider ENT review
Ongoing monitoring:
- OBS hourly for 4hrs
- Observe for headache, nausea, vomiting or changes in behaviour
- Observe for occult bleeding: flank pain, increased intra-abdominal pressure, swelling, bruising
- Review falls risk (now high) and risk prevention strategies
For reference: Sinus surgery
Indications for frontal sinus surgery:
Chronic rhinosinusitis e.g. ethmoidectomy
Cerebrospinal fluid leak
Benign and malignant tumours of the frontal sinuses
Complications:
Open/external: osteomyelitis, ocular muscle injury, haemorrhage, CSF leak, meningitis
Endoscopic: Epistaxis, bruising, dental pain, brain abscess, haemorrhage, CSF leak, meningitis
Post-op advice:
Avoid straining or blowing nose for 10days
Do not recommence aspirin or NSAID until 2wks post-op
Avoid steroid nasal spray for 2wks
Nasal saline sprays every 2-3hrs
Sinus irrigation twice daily
Normal to have intermittent blood discharge first 3-5days after surgery
You are working in emergency and you see a 4yr old boy with fever, left otalgia and acute left facial nerve palsy. What is the most likely diagnosis?
What condition could this develop into that causes swelling behind the ear and the ear to stick out/forwards?
The most likely diagnosis is Acute Otitis Media
• May present with otalgia, irritability, decreased hearing, anorexia, vomiting, or fever, usually in the presence of an ongoing viral respiratory infection.
o Risk factors: male, young age, exposure to respiratory viruses (day care attendance, older siblings),
positive family history, aboriginal
• Physical examination will reveal a bulging, opacified tympanic membrane with decreased mobility. The membrane may be white, yellow, pink, or red.
• Diagnosis is generally made with conventional otoscopy. Additional tests might include pneumatic otoscopy or tympanometry to confirm the presence of an effusion.
• Treatment includes pain control with analgesics and might include antibiotics.
• Complications include perforation of the tympanic membrane and, rarely, mastoiditis, seventh cranial nerve palsy, or sigmoid sinus thrombosis.
The condition that could develop causing swelling behind the ear and the ear to stick out/forwards is acute mastoiditis.
• There is oedema, erythema, and tenderness over the mastoid process.
• Because the mastoid air cells are connected to the distal end of the middle ear through a small canal or antrum, most episodes of AOM are associated with some inflammation of the mastoid
• Also managed with analgesia and abx
You are working in emergency and you see a 4yr old boy with fever, left otalgia and acute left facial nerve palsy. What is the most likely diagnosis?
How do you manage this boy?
The most likely diagnosis is Acute Otitis Media
Management:
• Oral analgesia
o Pain control is central to the management
o Paracetamol/ibuprofen
• Antibiotic therapy with penicillin (amoxicillin)
You are working in emergency and you see a 4yr old boy with fever, left otalgia and acute left facial nerve palsy. What is the most likely diagnosis?
What dangerous complications can result from this condition?
The most likely diagnosis is Acute Otitis Media Complications that can result include: • Intratemporal complications o Hearing loss o Balance and motor problems o TM perforation o Choleastoma o Mastoiditis
• Intracranial complications o Meningitis o Epidural abscess o Brain abscess o Lateral sinus thrombosis o Cavernous sinus thrombosis o Subdural empyema o Carotid artery thrombosis
You are the surgical ward resident. A nurse calls you saying that a patient who had frontal sinus surgery this morning had a fall in the toilet. They have also been bleeding intermittently throughout the day.
What will you do?
Urgently go to review the patient, concerned that patient may be hypovolemic due to post-op bloodloss
ABCDE
Response – patient conscious or unconscious, alert or obtunded
Airway – patent?
Breathing – RR, SaO2
Circulation: HR, BP, mucous membranes, 2x large bore IV cannulas with resuscitation fluids if needed,
review fluid balance chart
Disability - GCS
Review obs chart, medications (anticoagulants?), operation report, patient notes
Hx of fall
• Mechanism of fall? Injuries?
• LOC?
• Post-ictal period?
• Dizziess
• anticoagulation
• Events leading up to fall – dizzy? Lightheaded? Palpitations?
• Amount of bloodloss
• Past hx of bleeding disorders, seizures, IHD
• Review: chart, operative report, perioperative progress notes, adequacy of fluid replacement, omission
of important meds?
Examination:
• Repeat obs
• Examine site of bleeding
• Surgical site examination
• Neurological exam – focal defecits? Global defecits?
• Cardiovascular exam – including check for orthostatic hypotension
DDx – Hypovolaemia, aortic stenosis, seizures, arrhythmias, vasovagal/micturition syncope, MI, CVA, PE
Ix:
B – FBC, UEC, Coags, Group and hold
O –
X – X-ray for fractures
E – ECG for arrhythmias, MI
S – CT head looking for site of bleeding/ ?CVA if suspected/ head trauma
Mx:
Attempt to bring about haemostasis – upright position, packing if needed
Provide fluid resuscitation and treat any injuries as required
Contact ENT surgeon who performed operation for advice- urgent ENT review
Complications of functional endoscopic sinus surgery:
• CSF leaks, orbital injury, intracranial haemorrhage, meningitis
Important causes of post-operative collapse or rapid deterioration:
Cardiovascular MI, sudden arrhythmia, fluid overload, PE, stroke, haemorrhage
Respiratory Failed reversal of anaesthesia, drug induced respiratory depression
Surgical/infective Hypovolaemic shock- blood loss, decompensation, systemic sepsis, SSI
Metabolic Electrolyte disturbance, BGL abnormality, adrenal insufficiency –> hypotension
drugs Drug reaction e.g. anaphylaxis
A 65 yoa lady has presented with a lump in her right cheek, centred near the angle of the mandible. What is the differential diagnosis?
DDx - VITAMIN CDEF
Neoplasm (most likely): Parotid tumours Causes: Benign (70%) - Pleomorphic adenoma Malignant – Mucoepidermoid carcinoma, SCC, acinus cell carcinoma Hx: painless, firm, slow-growing mass Red flags: Red flags on Hx: Facial n. weakness, rapidly increasing size, ulceration of mucosa/skin Others: Lipoma (tumour of being adipose tissue) Non-Hodgkin Lymphoma Odontogenic Muscles of mastication
Infective/inflammatory: Parotid infection Bacterial (S.aureus, streptococci) Hx: sudden onset pain, swelling, purulence from duct, salivary stones Viral (Mumps, HIV, coxsackievirus) Hx: usually bilateral swelling, tenderness Odontogenic infection and abscess Hx: acute onset pain, swelling, trismus Lymphadenitis
Obstructive:
Sialolithiasis (salivary stone)
Hx: chronic/recurrent pain and swelling, occurs with meals, recurrent infections
O/E: swelling, milking gland produces saliva, intraoral palpable stone
Autoimmune:
Sjogren’s syndrome
Hx: xerostomia, keratoconjunctivitis sicca, abnormal taste, intermittent swelling
Idiopathic:
Sialadenosis (non-specific bilateral enlargement of the salivary glands)
How would you go about investigating a 60 year old lady who presented to you having found a lump, about 2cm across, in her left breast?
Approach: “Triple Test”
Positive if any component is indeterminant, suspicious or malignant
Sensitivity (true positive) 99.6%
Specificity 62% i.e. false positive 38%
Hx and O/E
Imaging (mammography and/or ultrasound +/- MRI)
Non-excisional biopsy (FNA +/- core biopsy)
Hx: HxPC Site (1/2+, localised or generalised) Onset/duration (rapid – cyst, slow – carcinoma) Growth or change (fluctuate with menstrual cycle – fibroadenoma, fibroadenosis) Previous lump Associated symptoms Pain/tenders (non-malignant) Nipple retraction or inversion Nipple discharge Skin changes (redness, swelling or puckering of the skin) Systemic features: Fever, weight loss, anorexia
PMHx/SHx
Previous breast problems (any previous Ix or results)
Obstetric history (RF for malignancy – late age at first pregnancy, low parity)
Menstrual history (RF for malignancy – early menarche, late menopause)
Obesity
Medications: the pill, HRT
FHx
Family history of breast/ovarian cancer – age of onset, bilaterality
O/E:
General inspection - patient sitting upright on side of bed
Asymmetry
Scars (small – lumpectomy, large diagonal – mastectomy)
Masses – size, location
Skin changes – erythema, puckering, peau d’orange (inflammatory breast cancer)
Nipple changes (retraction, discharge (green, blood-stained), scale)
Closer inspection - hands on hips (tenses pectoralis major), hand on head and lean forward
Palpation – hand behind head of breast you’re examining, including axillary tail and nipple
Location (quadrant of breast or clock face)
Size and shape
Consistency (smooth and round -benign, firm/hard – probable malignancy)
Edges (well-defined/irregular)
Mobility (mobile – benign, fixed – probably malignancy)
Fluctuance (fluid-filled – cyst)
Temperature
Overlying skin changes (erythema, ulceration, punctum)
General lymph node examination (axillary lymphadenopathy – probably malignancy)
What are the risk factors for breast cancer?
Non-modifiable: Increasing age (peak incidence 70-80s) Female Family or personal hx BRAC-1 or BRAC-2 mutation (5-10%) Menses <12y/o Late menopause >55y/o Previous breast biopsies showing non-malignant abnormalities
Modifiable: Obesity (fat tissue produces oestrogen) Alcohol (limits liver’s ability to control the levels of oestrogen and can damage DNA cells) Smoking First pregnancy >30y/o OCP/HRT Radiation exposure
Protective factors:
Regular physical activity
First pregnancy <20y/o
Breast feeding >12mths
Describe the options for adjuvant treatment for breast cancer.
Adjuvant therapy is given following primary surgical management to reduce the risk of local recurrence or prevent metastatic spread.
Radiotherapy
Indications:
Post breast-conserving therapy (lumpectomy)
Post mastectomy if high recurrence risk or nodal involvement
Delivered daily (except weekends) for 3-6wks via external beam to chest wall +/- local nodes
SE: Fatigue, skin reactions (sunburn), lymphedema, loss of appetite, predisposed to other cancers
Chemotherapy
Indications:
Lymph node involvement (regardless of size or menopausal status)
Premenopausal with invasive cancers (generally more aggressive)
HR -ve and HER2 +ve
Contraindications:
Less effective if node negative or post-menopausal
Non-invasive cancer
Combination: Anthracycline (doxorubicin) + Taxane (paclitaxel)
SE: nausea, vomiting, alopecia, neutropenia, fatigue, mouth ulcers, infertility
Hormone therapy
Indications:
ER +ve
Types:
Ovarian ablation (premenopausal)
GnRH agonists (supress ovarian function) or oophorectomy
Tamoxifen (premenopausal and low-risk postmenopausal)
Selective oestrogen receptor modulator that blocks peripheral action of oestrogen by binding to the receptor
SE: menopausal symptoms (vaginal dryness, hot flushes), clotting risk, endometrial cancer
Aromatase inhibitors (high-risk postmenopausal)
Prevents oestrogen synthesis in peripheral adipose tissues by blocking conversion of androgens to oestrogen by aromatase enzyme.
SE: osteoporosis, fractures, worsening menopausal symptoms
Delivered as daily oral medication for 5+ years post other treatments
Biological therapy
Indications: HER2 +ve
Trastuzumab (Herceptin)
Monoclonal antibody that targets HER2 receptors to downregulate action and initiate immune mediated destruction.
SE: flu-like, fever, anorexia, muscle/joint pain, diarrhoea
Delivered post-surgery and chemotherapy (must have or won’t be covered by PBS) at 3wk intervals for 1yr
What are the surgical options for a small confirmed cancer which is sited laterally in the breast, is not fixed to the skin, deep tissue or nipple? What are the advantages and disadvantages of the options?
- Breast conservation surgery
Background
Removal of tumour with margin of surrounding breast tissue + radiotherapy
Lumpectomy (1cm margins), wide local excisions (2-3cm margins), quadrantectomy
Indications Patient’s choice Single lesion, Stage I/II, <4cm Nil skin, nipple/areola involvement No node involvement CI: pregnancy, BRCA, SLE, lupus
Advantages Comparable survival Better cosmetic and psychological result Less invasive Shorter recovery
Disadvantages Follow-up radiotherapy Higher rate of recurrence Anxiety/fear of recurrence May require additions surgeries (poor margin or recurrence)
2. Mastectomy Background Simple/total: breast + level I nodes Modified: breast + lvl I-III nodes + pec minor Radical: modified + pec major
Indications Not a candidate for BCS Multicentric disease Positive margins after resection Radiation CI (pregnancy, recurrence of cancer)
Advantages Comparable survival Better cosmetic result if smaller breast Reconstruction potential Lower risk of recurrence No follow-up radiation if low risk
Disadvantages Comparable survival Better cosmetic result if smaller breast Reconstruction potential Lower risk of recurrence No follow-up radiation if low risk
What are the important differential diagnoses for a breast lump?
Malignancy: 10%
Non-invasive/pre-cancerous:
Ductal carcinoma in situ (DCIS) 80%
Lobular carcinoma in situ (LCIS) 12%
Invasive: Infiltrating ductal carcinoma (IDC) 75% Infiltrating lobular carcinoma (ILC) 10% Other: Medullary (5%), mucinous (colloid), inflammatory, tubular, Paget’s disease, papillary Metastases from another primary site
Benign:
Cyst (blockage of duct resulting in fluid-filled mass)
Hx: premenopausal/perimenopausal
O/E: compressible, ballotable solitary mass
Fibroadenoma
Hx: asymptomatic, incidental finding, <40y/o
O/E: smooth, rubbery, mobile mass
Fibrocystic change or fibroadenosis
Hx: breast pain, increase with menstrual cycles
O/E: not well-defined mass, nodular breast tissue
Fat necrosis
Hx: obese, large breast, blunt trauma, injection of substance, breast surgery, radiation therapy
O/E: skin ecchymosis, hard irregular lump
Breast abscess
Hx: lactating, mastitis, fever, malaise, painful mass
O/E: fluctuant, tender, palpable mass
Galactocele (milk retention cyst)
Hx: breastfeeding
Phylloides tumour
Hx: 40-60, rapid enlargement breast
O/E: well-delineated, large breast mass
Intraductal papilloma (local epithelial proliferation in mammary ducts) Hx: blood stained discharge Duct ectasis Hx: menopause, green nipple discharge
What is the influence of a patients age on the choice of imaging for breast cancer?
Indications:
>30 Mammography
<30 U/S or MRI if high risk (family hx, BRCA in first degree relatives, hx chest radiation)
Reasoning:
Denser breast tissue in younger women limits the sensitivity of mammography.
In patient <35 false-negative rate for mammography is >50%
MRI not affected by age or density of breast tissue
More sensitive, but not very specific which can result in unnecessary biopsies
A 26 yoa woman presents to the ED with a painful, hot, swollen left breast. She is 5 weeks post partum with her first child and has been breast feeding successfully. She was prescribed antibiotics for the problem 5 days ago by her GP but has been getting worse. She has no systemic previous medical history, takes no regular medication. On examination, she is febrile, and has a fluctant swelling in the left lower quadrant of her left breast with overlying erythema.
What is the diagnosis?
What is the treatment?
Dx: Lactational breast abscess
Mx:
Needle aspiration (+/- U/S guidance) or surgical incision/drainage.
Empirical abx (flucloxacillin) – modify based on culture of swab.
Pain relief – simple analgesia, ice pack.
Continue breast feeding/expressing milk – important for resolution of infection and relief of breast fullness.
? referral to lactation nurse consultant.
For reference: Lactational breast abscess
Definition: collection of infected fluid within the breast
Background:
20% breastfeeding women develop lactational mastitis, with 3% developing an abscess
Peak within first 6wks post-partum
Causative organism: S.aureus
Risk factors: hx mastitis, primiparity, maternal age >30, GA >41wks
Pathophysiology:
Milk stasis or milk overproduction coupled with bacterial invasion of the breast tissue through a traumatised nipple results in mastitis.
As the breast tissue becomes inflamed it further blocks milk ducts and allows for progression of the infection. Left untreated, it can cause tissue destruction resulting in an abscess.
Presentation
Hx: worsening mastitis unresponsive to abx
Breast pain, fever, flu-like (malaise, lethargy, myalgia, sweating, headache, nausea/vomiting)
O/E: Inflammation, fluctuant tender mass
How would you go about investigating a 16 year old girl who presented to you having found a lump, about 2cm across, in her left breast?
Dx: Fibroadenoma (most common lesion between 15-35y/o)
Reassure patient that most breast lumps benign, but must complete “Triple Test”
Triple testing is a combination of clinical examination, imaging and nonsurgical biopsy.
History:
HxPC
Site, onset, duration
Growth or change (fluctuate with menstrual cycle – fibroadenoma, fibroadenosis)
Previous lump
Associated symptoms (discharge, pain, tenderness, skin changes)
Systemic features (fever, weight loss, anorexia)
PMHx/SHx
Previous breast problems (any previous Ix or results)
Gynaecological, menstrural and obstetric hx
Medications – OCP
Hx cancer
FHx
Family history of breast/ovarian cancer – age of onset, bilaterality, BRAC genes
O/E:
Inspection: Asymmetry, scars, masses, skin changes, nipple changes/discharge
Palpation: Location, size, shape, consistency, mobility, temperature, tenderness/painless, skin changes
Lymph nodes
Ix:
U/S (well-defined, solid mass – fibroadenoma)
FNA or CNB (especially if red flags or suspicious imaging)
A 30 yoa woman presented with a painful lump in her left breast 5 days after childbirth. This is a photograph of the breast. What do you see?
Dx: Lactational mastitis +/- abscess
O/E:
Describe breast + quadrant
Nipple cracking (+/- retraction/inversion)
Localised area of inflammation (erythema, swelling)
Purulent discharge
Abscess punctum
Lactational localised to peripheral wedge-shaped areas of the breast (non-lactational central)
What is the most likely organism that might be causing an acute, lactation associated, breast abscess?
Staphylococcus Aureus
What is the management of an acute lactation associated breast abscess?
Needle aspiration (+/- U/S guidance) or surgical incision/drainage
Empirical abx (flucloxacillin) – modify based on culture of swab
Pain relief – simple analgesia, ice pack
Continue breast feeding/expressing milk – important for resolution of infection and relief of breast fullness
? referral to lactation nurse consultant
A 30-year-old woman presented with a painful lump in her left breast 5 days after childbirth. This is a photograph of the breast. What is the likely diagnosis and what is the management?
Dx: Lactational mastitis +/- abscess
Mx mastitis: Flucloxacillin 500mg 6hrly for 5days Effective milk removal Pain relief – analgesia, warm/cold compress Increased fluid intake
Mx abscess:
Needle aspiration (+/- U/S guidance) or surgical incision/drainage
Empirical abx (flucloxacillin) – modify based on culture of swab
Pain relief – simple analgesia, ice pack
Continue breast feeding/expressing milk – important for resolution of infection and relief of breast fullness
? referral to lactation nurse consultant
DDx:
Breast engorgement (occurs day 3-5 post-partum, requires frequent breast emptying)
Inflammatory breast cancer
A 50 year old woman is diagnosed with breast cancer. Her surgeon recommends a “lumpectomy” and sentinel node biopsy. The pathology report shows a 20 mm grade 3 ER positive, PR positive, Her-2 negative cancer, without lymphovascular invasion, and a negative sentinel node biopsy.
a) Describe three “adjuvant” treatments that would be potentially beneficial to her.
b) referring to the aims of each treatment and distinguishing between the aims of adjuvant systemic therapy and adjuvant local therapy.
For this patient I would recommend radiotherapy, hormone therapy and chemotherapy. I would not recommend biological therapy as she is HER2-ve and this would not be effective.
Radiotherapy
Indicated after breast conserving surgery
Aims to achieve local disease control by eradicating any residual disease and preventing its recurrence.
Hormone therapy (e.g. tamoxifen or aromatase inhibitors)
Indicated in hormone receptor positive cancers
Aims to reduce the level of circulating oestrogen and block hormonal stimulation of remanet cancer cells.
Chemotherapy
Indicated in HER2-ve grade 3 cancers >10mm
Aims to eradicate microscopic cancer cells that could recur as metastases
Most common regimen is Anthracycline + Taxane
Both local and systemic adjuvant therapies aim to improve survival by preventing disease recurrence. However, they can be distinguished by their mechanism of action. Local therapy acts directly on the breast to prevent recurrence of the primary tumour, systemic therapy acts through the whole body targeting metastatic spread or occult disease.
A 30-year-old man presents with a 3.2mm melanoma on his right arm. A wide excision and sentinel node biopsy is performed. One node is removed from the right axilla as part of the sentinel node procedure. It contains a 0.3mm deposit of melanoma. What are the biological arguments for and against proceeding to an axillary clearance?
For:
Staging and prognostic purposes
Determining appropriate adjuvant therapy
Reduce risk of further spread
Improves quality of life if bulky adenopathy in neck
Stage T3 or T4 (>2.0mm)
If macro metastases >2.0mm
Against:
No overall survival benefit (in comparison to observation alone)
May not prevent recurrence elsewhere
Approx. 80% of patients with positive sentinel node have no additional disease i.e. unnecessary surgery
Surgical risks – lymphedema, wound infections, nerve damage
Stage T1 or T2 (<2.0mm)
If micro metastases
Older patient, unstable patient, high peri-operative risk, life-shortening comorbidity
Definition: removal of all lymph nodes and possible tumour containing tissue from the armpit region.
Levels:
Lateral to pec minor
Up to medial border pec minor
Beyond medial border pec minor
A 53 year old man presents with a pearly nodule on his back that has, according to the patient “come up fairly quickly in the last 6 weeks”. It is bleeding when he rubs it with a towel. Q1. How should you mange this problem?
Dx: Nodular basal cell carcinoma
History: HxPC: Site, onset, duration Growth or change Association symptoms (pain, bleeding – after trauma) Risk factors: Previous skin malignancies Sun exposure (occupational/recreational) Exposure to radiation
O/E
Inspection:
ABCDE approach: (more for pigmented lesions)
Asymmetry, irregular borders, colour, diameter >6mm, evolution
Characteristic features:
Pearly/waxy, central depression, central ulceration, raised border, telangiectasias
Closer inspection via dermatoscope
Full skin check
Mx:
Surgical:
Excision biopsy (gold-standard) with >4mm margins – cure rate approx. 90%
Curettage and cautery – approx. 90%
Non-surgical:
Radiation therapy – cure rate approx. 90%
Cryotherapy – approx. 85-90%
Photodynamic therapy – approx. 80%
Topical immunotherapy (e.g. imiquimod) – approx. 80%
For reference: Nodular BCC
Definition: Locally invasive skin cancer
Risk factors:
Sun exposure
Fair skin
Previous skin cancer
Clinical features: Slow growing plaque/nodule Shiny/pearly nodule with smooth surface Variable size mm-cms Spontaneous bleeding or ulceration Rarely - rapid growing, invasive and/or metastatic to local lymph nodes
A 55 year old woman presents with a breast lump. Q1. What clinical and mammographic features together would allow you to be sure that it is benign?
Clinical features (alone not sufficient to differentiate):
Benign Multiple lesions “Rubbery” Mobile Well circumscribed border Bilateral, milky discharge Fluctuating
Malignant Single lesion Hard Immobile Irregular borders Unilateral, bloody, clear or coloured discharge Skin changes Hx of cancer or systemic symptoms
Mammographic features:
Benign Round/oval shape Well circumscribed border Low density Large, coarse (‘popcorn-like’) or smooth, round calcifications
Malignant Stellar shape Irregular (spiculated) border High density Clustered or linear branching micro-calcifications
A 45-year-old woman presents with a new breast lump that she has only just noticed. What single investigation might allow you to reassure her, and what finding on that investigation would be most reassuring?
Gold-standard Ix is the triple test, as no test alone has greater sensitivity or specificity.
In terms of a single most reassuring investigation, it would be a core biopsy for cytology and histopathology. Although, this usually requires imaging first.
Reassuring findings:
Histology of tissue specimen being void of malignant change (e.g. differentiated or irregular cells, abnormal proliferation)
A 36-year-old woman has a mastectomy and axillary clearance for a large “triple negative” breast cancer with multiple nodes involved. She is offered adjuvant chemotherapy which should provide a relative risk reduction of about 25%. Without treatment, this woman’s chances of 5-year survival are 20%. What are her chances of survival with the adjuvant chemotherapy? Explain how you arrived at this answer.
Relative risk reduction: relative/proportional risk reduction of an event happening in 2 groups with different interventions
Current risk of death in 5yrs is 80%, and chemotherapy offers a 25% reduction relative to that risk
i.e. new risk of death is 0.25*0.8 = 0.6 = 60%
So, if the new risk of death is 60% with chemotherapy, the new chance of 5yr survival with chemotherapy is 40%
A 50-year-old woman has a wide excision and sentinel node biopsy for a breast cancer. The pathology report shows a 30mm grade 3 ER positive, PR positive, Her-2 negative cancer, without lymphovascular invasion, and a negative sentinel node biopsy. Describe the adjuvant treatment options available to her. Refer to the different aims of adjuvant systemic therapy and adjuvant local therapy.
For this patient I would recommend radiotherapy, hormone therapy and chemotherapy. I would not recommend biological therapy as she is HER2-ve and this would not be effective.
Radiotherapy (via external beam)
Indicated after breast conserving surgery
Aims to achieve local disease control by eradicating any residual disease and preventing its recurrence.
Hormone therapy (e.g. tamoxifen or aromatase inhibitors)
Indicated in hormone receptor positive cancers
Aims to reduce the level of circulating oestrogen and block hormonal stimulation of remanet cancer cells.
Chemotherapy
Indicated in HER2-ve grade 3 cancers >10mm
Aims to eradicate microscopic cancer cells that could recur as metastases
Most common regimen is Anthracycline + Taxane
Both local and systemic adjuvant therapies aim to improve survival by preventing disease recurrence. However, they can be distinguished by their mechanism of action. Local therapy acts directly on the breast to prevent recurrence of the primary tumour, systemic therapy acts through the whole body targeting metastatic spread or occult disease.
How are anal fissures treated?
Conservative:
General measures:
Avoid straining
Mx constipation – Fibre, stool softeners e.g. Movicol, fluids
Sx analgesia
Warm anal bathing – stimulates somatoanal reflex
Topical ointment (1st line)
GTN – smooth muscle (sphincter) relaxation and vasodilation
CCB (diltiazem) – blocks smooth muscle contraction
Botulinum toxin injection (2nd line)
Sphincter relaxation (paralysis) for 8-12wks
Surgical:
Lateral internal sphincterotomy
Indicated post 6-8wks of conservative therapy
Method: Divide 1cm of the lower portion of the internal sphincter
Risks: incontinence of flatus or stools, infection
What is an anal fissure? How does it develop? What causes it to persist?
Definition: Longitudinal tear in the mucosa and skin of the anal canal distal to the dentate line. Approx. 90% occur in the posterior midline, whereas secondary causes more associated with lateral tears.
Aetiology: Acute Primary: Passage of hard stools Explosive diarrhoea Vaginal delivery Foreign body penetration Secondary: IBD Malignancy Infection (syphilis, HIV) Chronic Approx. 40% develop into chronic form due to: Repeated tearing “Fear of defecation” which aggravates constipation and straining Impaired healing If extends into internal anal sphincter (involuntary smooth muscle) it can lead to spasm and reduced blood flow with ischemia. Exposure to faeces also slows healing
Presentation: Pain post defecation Tearing, ripping, burning character Minutes to hours Hx constipation Irritation or itchiness around skin Haematochezia Fissure or small skin tag (sentinel pile) at 6 or 12 O’clock position PR exam extremely painful
How would you go about investigating a previously well 63-year-old man who presented to you with a short history of rectal bleeding?
DDx: Colorectal Ca Benign polyps Haemorrhoids, fissures IBD Diverticular disease Infectious (gastroenteritis)
Hx: PC Bleeding Onset, duration, previous bleeds, mucous, colour Bright red – anorectal Dark red, clots – rectosigmoid Dark red mixed in stool – ascending/transverse Melena – Upper GI Associated Sx Pain, weight loss, fever, night sweats, changes in bowel habits PMHx Any of the DDx Abdominal surgery Hx Coagulopathies (incl. medications) RF for Colorectal Ca Polyposis syndromes (FAP, HNPCC, juvenile polyposis) FHx PMHx polyps or CRCa Chronic UC or Crohn’s Diet poor in fruit and vegetables Smoking/alcohol Long term NSAIDs
O/E
Vitals
Abdo (conjunctival pallor, palmar pallor)
PR (haemorrhoids, anal fissure, abscess, distal carcinoma or polyp)
All patients >45 with new onset rectal bleeding must be investigated due to high risk of CR Ca
Ix:
Pathology
FBC, UECs, coagulation studies, LFTs (metastatic disease)
Bedside rigid sigmoidoscopy
Visualised up to splenic flexure (75% of CR Ca)
Note that most PR bleeding with CR Ca will be distal to descending colon
Colonoscopy +/- biopsy
Other:
? Upper GI endoscopy
If CR CA suspected - CT colonography, abdominal CT
How would you go about investigating a previously well 63-year-old man who presented to you with a short history of anaemia?
DDx:
Microcytic
Decreased intake (lack of red meat ingestion)
Blood loss (CR Ca, haemorrhoids, IBD, menorrhagia, PUD)
Decreased absorption (surgery, Crohn’s disease)
Increased demand (puberty, pregnancy)
Normocytic
Increased reticulocytes i.e. Hyperproliferative
Haemorrhage
Haemolytic (thalassemia, sickle cell, autoimmune, infection, drugs)
Decreased reticulocytes i.e. hypoproliferative
Aplastic, leukaemia
Macrocytic Megaloblastic (hyper segmented neutrophils) B12 deficiency Folate deficiency Non-megalobalastic Alcohol Liver disease
Hx: PC Source of bleeding? (GIT, bladder, menstural) Previous Hx Diet (iron, folate, B12) PMhx IBD, coeliac, GORD, malignancy Coagulopathy Surgical Hx (gastrectomy, bariatric) Medications NSAIDs, steroids Antiplatelet, anticoagulant Smoking, alcohol, drugs
O/E:
Vitals
Abdo (conjunctival pallor, SOB, mass, hepatomegaly etc.)
PR
Ix: Pathology FBC RBC <130 in M OR <120 W Microcytic 🡪 iron studies Normocytic 🡪 reticulocytes Macrocytic 🡪 blood film + serum B12/folate Blood film/peripheral smear Cell morphology e.g. sickle cell Iron, B12, Folate UECs, LFTs Other: If occult blood – FOBT, colonoscopy If haemoptysis/melena – upper GI endoscopy If cause unclear – bone marrow biopsy (hypo proliferative normocytic anaemia)
How would you go about investigating a previously well 63-year-old man who presented to you with a short history of a change in bowel habit to more frequent evacuation of loose stools with mucous?
DDx: IBS, IBD (especially UC) CR Ca Haemorrhoid, fissure, villous adenoma Bacterial infection Obstruction, colostomy
Hx: PC Bowel habits – duration, frequency, Bristol stool chart Associated Sx – bleeding, abdo pain, n/v/a, weight loss, fever Sick contacts, recent travel Previous Hx PMHx Any DDx RF CR Ca Polyposis syndromes (FAP, HNPCC, juvenile polyposis) FHx PMHx polyps or CRCa Chronic UC or Crohn’s Diet poor in fruit and vegetables Smoking/alcohol Long term NSAIDs Previous Ix Medications Abx – Clostridium Difficile
O/E:
Vitals
Abdo + PR
Ix: Pathology FBC UECs Electrolyte disturbances from diarrhoea) – hypokalaemia, hyponatremia CRP, ESR Nutritional deficiencies (B12, vitamin D) Stool culture + MCS FOBT Consider: Rigid/flexible sigmoidoscopy Colonoscopy +/- biopsy Upper GI endoscopy +/- biopsy Barium studies (IBD) Contrast CT (localise extent, assess mural and extra mural involvement)
What are the major risk factors for colorectal cancer?
Epidemiology: CR Ca 2nd commonest tumour, most common GI malignancy Approx. 1/12 Australians M>F, Peak: 55-75y/o
RF: Modifiable Diet (high red meat, low fibre (fruit/vegetables)) Obesity Physical inactivity Smoking Heavy ETOH Long term NSAIDs Non-modifiable Age >50 FHx 1# PMHx polyps or CRCa IBD Hereditary polyposis syndromes (FAP, HNPCC – lynch syndrome), juvenile polyposis)
What method is used for population screening for colorectal cancer in Australia?
Screening: Low Risk Asymptomatic, Nil FHx Method: FOBT Two stool samples on consecutive days to assess for blood 50-74y/o every 2 years 70% sensitivity (i.e. 30% false negatives) Moderate Risk 1# relative OR 2x2# relative Method: FOBT – 2yearly Colonoscopy – 5yearly or 10yrs before FHx diagnosis High Risk 3x1# relative, extensive premature adenomas Method: Refer to bowel cancer specialist ? Flexible sigmoidoscopy or colonoscopy 1-2yearly >25y/o or 5yrs before affected family member Genetic screening Symptomatic Colonoscopy not FOBT
What is neoadjuvant therapy for rectal cancer?
Neoadjuvant: Definition: Treatment given prior to definitive surgery to shrink tumour making it more easily resectable Initiations: T3/T4 T1/T2 with nodal involvement Distal rectal tumour where APER is required Tumour that invades mesorectal fascia Benefits: Higher sphincter preservation Improved resectability Tumour regression Therapy: Conventional fractioned radiotherapy +/- chemotherapy (fluropyrimidine) Long course approx. 7-10wks All pts who receive neoadjuvant also require 4m of adjuvant chemotherapy
Definitive:
Surgical resection +/- lymphadenectomy
Site of cancer Type of resection Anastomosis Caecal, ascending or proximal transverse colon Right hemicolectomy Ileo-colic Distal transverse, descending colon Left hemicolectomy Colo-colon Sigmoid colon High anterior resection Colo-rectal Upper rectum Anterior resection (TME) Colo-rectal Low rectum Anterior resection (Low TME) Colo-rectal (+/- Defunctioning stoma) Anal verge Abdomino-perineal excision of rectum None
Palliative For unresectable metastases or tumours Method: Chemotherapy Obstructing tumours may be endoluminally stented with self-expanding metal stents or transanally ablated if rectal
Where are the common sites that metastatic disease from a colorectal primary are found?
Metastatic:
Background
Approx. 15-20% have metastasis at time of diagnosis
Location
Lymphatic spread
Lower rectum – internal iliac
Middle/upper rectum – pararectal, inferior mesenteric
Descending/sigmoid colon – inferior mesenteric
Transverse/ascending colon – superior mesenteric
Haematogenous spread
70% Liver (via portal venous system)
35% Lungs (via venous spread)
21% Peritoneum
Other: Bone, ovaries, kidneys, brain
Distal extension
Outer bowel wall direct contact – liver, greater curvature of stomach, duodenum, small bowel, pancreas, spleen, bladder
Rectal cancer – bladder, prostate, vaginal wall, sacrum
Staging Ix:
Assessment of metastases (liver, lung or para-aortic)
Thoracoabdominopelvic CT
CT PEP scan to evaluate equivocal lesions
Assessment of local extent
Colonic – CT adequate
Rectal – Pelvic MRI and TRUS
Assessment of synchronous tumours
Colonoscopy or barium enema to confirm no secondary tumour
Tumour marker
CEA not useful for diagnosis or staging but can indicate relapse if high at time of diagnosis and falls to normal after resection
What are the treatment options for liver metastases from a colorectal primary?
Pathological staging: Dukes (approx. %5yr survival) A – confined to bowel wall only (90%) B – through bowel wall (70%) C – any with +ve lymph nodes (50%) D – any with metastases (<10%) TNM T1-4 – stages of invasion of bowel wall N0/1/2. No/up to 4/more than 4 lymph nodes involved M0/1, Metastases not present/present
Confirming Metastases Imaging CT FDG-PET Biopsy FNA Laparoscopic needle biopsy
Regional Mx:
Surgical resection (Approx. 50% 5YS)
Eligibility discussed by MDT
Patient factors – fitness for surgery, preference
Cancer factors
No extrahepatic metastasis
No portal vein, hepatic artery or bile duct involvement
Adequate hepatic function post resection (2/8 segments)
Radiofrequency ablation (14-55% 5YS)
Insertion of image guided probe percutaneously into tumour which emits high energy electrical currents effectively burning cancer
If not eligible for surgery
Selective internal radiation therapy (SIRT)
Injection of tiny radioactive beads providing specific radiation to liver
Trans-arterial chemoembolization
Chemotherapy injected directly into liver vessels
Based on principle that hepatic cancers preferentially receive blood supply from hepatic artery whilst normal parenchyma are supplied predominantly by the portal vein
Systemic Mx: Chemotherapy Neoadjuvant Transitions 10-15% of unresectable to resectable Adjuvant Post-surgery for 6m Palliative 10% 5YS, prolongs survival 5-20months Decrease symptoms of jaundice, pain, nausea, fatigue
What is the Adenoma-Carcinoma Sequence?
Background:
Process by which mutational activation of oncogenes and inactivation of tumour suppressor genes result in cancer.
In CR Ca, this is the process in which benign polyps undergo dysplastic change to become malignant adenocarcinomas.
Process takes approx. 5-10years
Pathophysiology: Pathway of mutation Chromosomal instability (85%) Microstallite instability (15%) Genes: Oncogenes: KRAS Tumour suppressor genes: APC and p53 Progression Normal mucosal cells of intestine continuously lost into lumen due to apoptosis and exfoliation, and continually replaced by proliferation at the crypt base. Subsequent to mutations it progresses from normal to low-grade dysplasia to high-grade dysplasia to carcinoma
CR Ca:
Predominant type adenocarcinoma
May occur as a polypod, ulcerating, stenosing, or infiltrative tumour mass
Majority (75%) lie on left side of colon and rectum
What is Crohn’s Disease?
Definition: Chronic immune-mediated inflammatory disorder that can affect any part of the GIT. Most commonly small/large bowel, particularly terminal ileum and right colon.
Epidemiology:
>incidence the further from the equator
Peak 15-30y/o and 55-80y/o
Idiopathic
RF:
Smoking
Sx:
RIF Abdo pain + diarrhoea (+/-blood/mucus) + weight loss
Constitutional: malaise, fever, n/v/a
Extra intestinal: conjunctivitis, iritis, gallstone formation
O/E:
Abdo
Inspection: wasting, anaemia, red fat tongue, oral ulcers
Palpation: tenderness, RIF mass (when omentum wraps around inflamed terminal ileum)
PR: nodules, perianal fissures
Ix:
Diagnosis: Colonoscopy (most sensitive) + Biopsy
Consider:
Pathology
FBC, ESR, CRP, Iron/B12
Decreased Hb and albumin (protein-losing enteropathy)
Stool MCS + culture – exclude infection (e.g. C.Difficile)
Antibody tests to distinguish from UC
Imaging
Abdo CT
Assess strictures, disease location, wall thickening, complications
Small bowel contrast study (barium enema or from duodenum via NG tube)
Narrowing of lumen, fibrosis, nodularity, cobblestones, ulceration
Gross Appearance:
Scope
Pink (preserved mucosa) between Yellow/white (ulcers) – “Skip lesions”
Fissures extending from epithelium into muscle wall – “Cobblestone appearance”, abscess, fistula, strictures
Transmural inflammation – thickened bowel wall
Fat wrapping (fingers of mesenteric fat wrap around the bowel due to chronic inflammation)
Histological
Non-caseating granuloma (approx. 50% of biopsies)
Lymphoid aggregates
Transmural inflammation
Submucosal oedema
Fibrosis
Cx: Fissures Fibrosis – Strictures, bowel obstruction Fistulas Adhesions Perforation – Peritonitis
Fistula’s:
Approx. 35% affected
Commonly involve small bowel
Mx:
Regression with aggressive medical mx
Resection of affected bowel segment + closure of other viscera
Note: high risk of leak due to impaired healing process in CD
Perianal disease: Approx. 20% Involves fissures, abscesses and fistulas Presentation: Devastatingly painful Bleeding Mx: Fissures: Sitz baths, stool softeners, analgesics Abscess: incision and drainage under GA Fistula: Surgical Setons (rubber band threat through fistula to keep it open and allow drainage preventing infection) Fistulotomy Bowel resection Abx cover
Medical Mx:
Anti-inflammatory agents
5-ASA (Sulfasalazine and Mesalazine) – first-line acute and long-term
Corticosteroids (prednisolone, budesonide) – acute exacerbations
Immunomodulators
Azathioprine, 6-mercaptopurine
Infliximab (anti-TNFalpha antibodies)
Dietary modification
Elemental diet to reduce inflammatory factors
Liquid/low fibre diets if at risk of obstruction
Supplemental calories, iron and vitamins
Surgical Mx: Approx. 50% end up with surgery Indications: Acute complications – fistula, obstruction, abscess, fulminant colitis, toxic megacolon Failure of medical therapy Cancer Growth retardation
What is Diverticular Disease?
A diverticulum is an outpouching of a hollow or fluid filled structure.
Diverticular disease is defined as clinically significant and symptomatic diverticulosis due to diverticular
bleeding, diverticulitis, segmental colitis associated with diverticula, or symptomatic uncomplicated
diverticular disease.
Diverticular disease is extremely common, affecting more than 50% of people over age of 70.
Common sites affected:
Potential defects can occur between longitudinal muscle bands (teniae coli) where mucosal vessels
penetrate the wall from the outside
• Sigmoid colon most commonly affected
• Right sided diverticular- rare but often ddx for appendicitis
• Isolate caecal diverticula - common in Western countries- susceptible to perforation
The pathophysiology is likely related to functional hypersegmentation - lack of dietary fibre –> high
intraluminal pressure –>mucosa herniate through muscle layers at weak points of intestinal wall.
Aetiology: dietary fibre deficiency, obesity, red meat consumption, NAIDS, coffee and alcohol, steroids,
decreased physical activity, altered colonic wall structure
What is Diverticulitis? How Does it Present Clinically?
Diverticulitis is the inflammation of a diverticulum. It be acute or chronic, uncomplicated or complicated.
Jessica Redmond 81
Usually presents with continuous LIF pain, altered bowel habits, nausea and vomiting, flatulence and
bloating, and is systemically unwell with fever, tachycardia leucocytosis in symptomatic acute diverticulitis.
It can also present with sign of perforation (peritonism, distended tympanic abdomen, reduced/absent
bowel sounds), and signs of fistula (PV discharge, UTI)
CT is imaging of choice.
Management involves bowel rest, antibiotics (amoxicillin, met, gent, tazocin) and surgical intervention for
complications in particular such as perforation, fistula, abscess.
Uncomplicated
-bowel rest
-abx (cipro + met)
-high fibre diet
-colonoscopy 6 weeks post to exclude colon cancer
Complicated
Empirical Abx - taz or ceftriaxone and met
Mx complications - surgical - colonic resection etc.
What are the Major Complications That can Result From Diverticulitis?
The major complications of diverticulitis are:
• Diverticular abscess
o Pericolic abscess often presents with persistent pain and tenderness, swinging pyrexia, incomplete
obstruction, or even sepsis
o Dx made on CT
o Abx if <4cm, or >4cm drained percutaneously under radiological guidance
• Fistula
o Can occure when inflammed diverticlum lies close to another hollow viscus
o Vesicolic fistula = pneumaturia and severe UTI
o Colo-ileal fistula = diarrhoea
o Colovaginal - PV discharge
o Surgical intervention
• Bowel obstruction
o May occur due to relative luminal narrowing from pericolonic inflammation or compression from
abscess
• Free perforation
o Often from abscess
• Segmental colitis
• Acute PR haemorrhage although not common in diverticulitis as much as diverticulosis
What are haemorrhoids and how are they classified or described?
Haemorrhoids are normal vascular structures in the anal canal, arising from a channel of arteriovenous
connective tissues that drains into the superior and inferior haemorrhoidal veins. They can enlarge and
thus become symptomatic. The cardinal features of haemorrhoidal disease include bleeding, anal pruritus,
prolapse, and pain due to thrombosis.
Constipation and pregnancy are the most common triggers for their development . Straining raises intra-
abdominal pressure which obstructs venous return, causing venous plexuses to engorge, they are dragged
distally by hard stool.
Haemorrhoids are classified as either internal or external, or mixed. External haemorrhoids are distal to the
dentate line in the anal canal and are covered by skin. Internal haemorrhoids are proximal to the dentate
line, and are classified into 4 grades based on extent of prolapse.
Classification of haemorrhoids:
Grade I: never prolapse- protrusion limited within anal canal
Grade II: prolapse during defecation and then return spontaneously
Grade III: piles remain outside anal margin unless replaced digitally.
Grade IV: irreducible and may strangulate
Jessica Redmond 82
Symptoms and signs of haemorrhoids:
• Attacks can last days-weeks, intermittent symptoms
• May bleed from stool trauma during defecation
• May prolapse and thrombose- acute severe pain
• Atrophy- by thrombosis and fibrosis –> skin tags at anal margin
• Strangulated haemorrhoids
How are Haemorrohids Treated?
Treatment is generally conservative, medical or surgical.
Conservative: (all)
• High fibre diet to reduce constipation
• Squatting or feet up to reduce straining
• Avoid straining and excessive time on toilet
• Moist gentle wiping
• Adequate fluid intake
Medical:
Creams, suppositories and topical preparations
• Grade 1- hydrocortisone cream short term for pruritus and pain
• Anaesthetics - benzocaine rectal ointment
• Stool softeners to decrease straining
• antispasmodics
Surgical
• Sclerotherapy - grade 2+
o Indication: First degree haemorrhoids that don’t regress with dietary, and secondary
o Outpatient
o Injection of sclerosing agent with proctoscope
o Injected into submucosal tissue (NOT the haemorrhoid itself) fibrotic action rapidly obliterating the
haemorrhoidal vessels.
o Usually repeated on 2-3 occasional 4-6 weeks apart
o not for people with nut allergy
• Banding - grade 2+
o Cone around base of haemorrhoid to constrict the vessels
o Uses anoscope
o Ischaemic necrosis occurs distal
o l to bands and they fall off
• Haemorrhoidectomy (grade 4+, external)
o Indications 3rd degree haemorrhoids
o Mass excised with overlying skin and mucosa. Secondary intention healing
o Stapled - sometimes for large
What are the Indications for Surgery in a Patient with Ulcerative Colitis?
• Colectomy indications:
o Patients with persistent symptoms
o Chronic disabling symptoms of intractable diarrhoea, urgency, recurring anaemia, failure to maintain weight and nutrition
o Children with failure to thrive
o Patients with increased cancer risk
o Patients with longstanding colitis
o Patients with dysplastic or adenomatous polyps
Most patients undergo a restorative proctocolectomy with ileal pouch anal anastomosis
What are the Indications for Elective Surgery in a Patient With Crohn’s?
- Intra-abdominal, retroperitoneal, abdominal wall abscess
- Symptomatic fibrotic stricture causing intestinal obstruction
- Enteric fistula
- SB or CRC from longstanding Crohn’s
- Intractable disease
What are the Indications for Emergency Surgery in a Patient With Inflammatory Bowel Disease?
For emergency or urgent indications, a total abdominal colectomy with end ileostomy is typically
performed.
• Colectomy indications:
o Toxic megacolon
o Perforation
o Major haemorrhage
o Urgent rx for fulminant cases failing to respond to intensive medical management.
The rectum is not resected but left as a defunctionalised Hartmann’s pouch with or without a mucus fistula.
In patients who develop significant rectal bleeding after total abdominal colectomy, completion
proctectomy with an ultralow Hartmann’s pouch or transanal suturing of the bleeding rectal ulcers can be
performed to achieve haemostasis. After the patient is stabilized, a completion proctectomy with
reconstruction can be performed at a later time.
You are the ED intern on duty. You are asked to see a 20 yoa man who has presented with perianal
pain that started 24 hours ago and has got worse despite appropriate analgesia. The pain is worse when
he walks or sits.
This has not happened before. He has no other relevant history. On examination you find a fluctuant,
warm, red 3cmx3cm swelling at the anal margin. What is the likely diagnosis?
The most likely diagnosis is a perianal abscess.
Perianal abscess - originates from an infected anal crypt gland
• Perianal abscesses transverse distally in the intersphincteric groove into the perianal skin where they
present as a tender fluctuant mass
• If undrained, a perianal abscess can expand into adjacent tissues
(e.g. ischiorectal space, supralevator space) and progress to sepsis
Epidemiology: mean age 40 years (but any age from 20-60), M:F = 2:1
Pathogenesis: A perianal abscess usually originates from an infected anal crypt gland– the glands penetrate
the internal sphincter and end in the intersphincteric plane.
An abscess develops when an anal crypt gland becomes obstructed permitting bacterial growth and
abscess formation
An anal fistula is a connection between two epithelial lined structures, and connects the anal abscess to
the skin of the buttocks
Clinical features:
• Severe pain in anal/rectal area
• Pain may be constant or associated with walking/sitting
• Constitutional symptoms such as fever, malaise
• Purulent discharge if abscess has started to drain spontaneously
• Erythema, lump
• May be no findings of inspection- abscess felt upon DRE
Approximately half of anal abscesses will result in the development of a fistula from the
inciting anal gland to the skin
DDx:
• Supralevator abscess, ischiorectal abscess, intersphincteric abscess
• Anorectal fistula- epithelialsed track between the abscess and perirectal skin or an adjacent organ e.g.
bladder
Jessica Redmond 84 • Internal haemorrhoid • Pilonidal abscess Ix: B –FBC, abscess culture if starting antibiotics, blood cultures if septic O –PR exam X – E S –CT if suspicious of nonpalpable abscess Mx: • Incision and drainage • Antibiotics -especially in valvular heart disease, immunosuppression, cellulits, diabetes
You are the ED intern on duty. You are asked to see a 20 yoa man who has presented with perianal
pain that started 24 hours ago and has got worse despite appropriate analgesia. The pain is worse when
he walks or sits.
This has not happened before. He has no other relevant history. On examination you find a fluctuant,
warm, red 3cmx3cm swelling at the anal margin.
What is the likely diagnosis?
What aetiological factors are associated with this condition?
Perianal abscess
• Most result from infection of the anal glands with colonic bacteria (cryptoglandular infections)
o The anal canal has 6-14 glands that lie in the plane between the internal and external anal sphincters
o Ducts from these glands pass through the internal sphincter and drain in to the anal crypts at the
dentate line
o This glands may become infected when the crypt is occluded by impacted faeces, oedema (from anal
trauma e.g. passage of hard stool) or as a result of inflammation e.g. Crohns
Aetiological factors/ risk factors:
• Male (M:F = 2:1)
• Crohn’s disease (1/3 of patients)
• Hard stools
• Age 20-60 (mean 40)
What is Ulcerative Colitis?
Ulcerative Colitis is an inflammatory disorder of the mucosa and submucosa of the large bowel. It is
characterised by recurrent acute exacerbations and intervening periods of quiescence or chronic low grade
activity.
Epidemiology
• Typically a young adult with a history of several weeks of frequent loose stools, later streaked with
blood and mucous.
• Attack often starts with attack of gastroenteritis or traveller’s diarrhoea that fails to settle
Aetiology
• Unclear
Jessica Redmond 85
Pathophysiology
• the disease always involves the rectum and extends proximally
• Can processed to backwash colitis in pancolitis patients
• Colonic mucosa becomes acutely inflamed –> neutrophil accumulation –> crypt abscesses
• Sloughing of mucosa –> small superficial ulcers
• Dysplastic changes occur over time –> prolonged or repeated episodes of inflammation can cause
progression to adenocarcinoma
o 5% after 10 years of colitis
o Aggressive
o On average occurs 20 years earlier than general population
• Lead pipe colon - fibrosis from longstanding colitis causing smoothing of haustrations an shortening of
colon
Clinical Features
• Acute attacks:
o Loose blood stained stools streaked with mucous
o Diarrhoea severity corresponds to extent of inflammation
• May pass 20+ stools/day
• Preceded by abdominal cramps
o Fulminant colitis
• Severe dehydration and electrolyte imbalances and blood loss
• Subgroup of patients with severe ulcerative colitis who have more than 10 stools per day,
continuous bleeding, abdominal pain, and distension, and acute, severe toxic symptoms
including fever and anorexia.
o Weight loss
o Extra-colonic features tend to flare up in parallel with colitis, except liver-related ones.
o Tenesmus
• O/E
o PR exam, proctoscopy and sigmoidoscopy
Extra-colonic features:
• Anaemia
o Normochromic normocytic- typically chronic and non specific
• Arthropathy
o Sacroillitis/ankylosing spondylitis, enteropathic arthropathy
• ~20% cases
• Anterior uveitis
o ~10% cases
• Primary sclerosing cholangitis
o Progressive fibrosis of intrahepatic biliary system –> cirrhosis, progressive liver failure, jaundice,
death
o Other liver presentations: chronic active hepatitis, bile duct carcinoma
• Erythema nodosum or pyoderma gangrenosum (purulent skin ulcers)
Complications:
• Toxic megacolon
o Colon dilates massively and patchy necrosis eventually occurs. The patient is systemically ill with
high fever, marked tachycardia and dehydration.
o Perforation and fatal peritonitis required emergency colectomy
o Causes:
• Inflammatory: UC, Crohn’s
• Infectious: bacterial (C diff, salmonella, shigella, campylobacter), parasitic, viral,
pseudomembranous colitis
The diagnosis of toxic megacolon should be considered in all patients presenting with abdominal
distension and acute or chronic diarrhoea. The diagnosis is clinical, based upon the finding of an
enlarged dilated colon on abdominal imaging accompanied by severe systemic toxicity.
o Medical therapy is the first line of treatment for patients with IBD and toxic megacolon and
includes fluid resuscitation and correction of laboratory abnormalities, administration of broad-
spectrum antibiotics, intravenous corticosteroids, complete bowel rest, bowel decompression with
a nasogastric or long intestinal tube, and a surgical consultation.
o The operative procedure of choice is a subtotal colectomy with end-ileostomy and is reserved for
patients who do not improve on medical management.
Investigations
• B: FBC, UEC, LFT, CRP, ESR
o Raised inflammatory markers, anaemia, hypoalbuminemia
• O: stool samples: MCS, OCP, C Diff
• X: barium enema usually not in acute disease, AXR monitoring dilatation as a sign of toxic megacolon -
likely do a CT tbh
• E
• S: proctoscopy, sigmoidoscopy, biopsy, colonoscopy in non acute setting
Ddx
• Crohn’s disease
• Radiation colitis
• Infectious colitis
• Diverticulitis
• Ischaemic colitis
• Vasculitis
• IBS
Management
Management depends on the severity of an individual attack, amount of colon involved, extent of chronic
symptoms and risks of long term complications.
Medical
• 5-ASA
o Mild attacks of proctitis/procto-sigmoiditis treated with local 5-ASA suppositories or enemas
o Mild attack of pan colitis with orals
o Avoid aspirin and NSAIDs as exacerbate despite being related
• Corticosteroids
o Suppositories or enemas used for local rx of rectal inflammation
o Short course high dose oral for severe exacerbations, IV if seriously ill
• No evidence for bowel rest and TPN
• Immunosupressives
o Ciclosporin and azathioprine
• settles on steroids but flares as weaned off steroids
• Maintenance therapy recommended in all patients with left sided colitis, pancolitis, extensive colitis.
• Supportive:
o Avoid anti-diarrhoeals e.g. loperamide - precipitate toxic dilatation
o High calorie and protein diet to counteract losses
Surgical
• Only needed in ~20% patients
• Colectomy indications:
o Urgent rx for fulminant cases failing to respond to intensive medical management
o Acute cases which progress to toxic megacolon, perforation or major haemorrhage
o Chronic disabling symptoms of intractable diarrhoea, urgency, recurring anaemia, failure to
maintain weight and nutrition
o Children with failure to thrive
o Patients with longstanding colitis who develop dysplasia or malignancy
• Total colectomy is curative
• Three main options:
o Subtotal colectomy with ileostomy
• Safest in emergency situation in sick patient on high dose corticosteroids
• Patient left with inflamed rectal stump
• Can change to other option when patient is well
o Proctocolectomy with permanent ileostomy
• Includes removal of rectum
• Generally recommended for elderly patients with sphincter preserving procedures are
inadvisable
o Restorative proctocolectom- ileoanal pouch
• Sphincter preserving operation avoiding permanent ileostomy
• Pouch from terminal ileum anastomosed into upper anal canal
Health maintenance:
• Immunisations
o Live vaccines contraindicated in immunosuppressed people
• Screening for cancer
o CRC, skin, cervical - skin because therapies
• Screening for anxiety and depression
• Prevention and monitoring of bone loss
o Increased risk, especially if on glucocorticoids
• Lab monitoring of complications of IBD
o Anaemia, b12 and folic acid deficiency
• Medication side effect monitoring
This lesion is found at colonoscopy in the proximal colon. What is it? How should it be treated? Is
follow up or surveillance needed?
If Polyp:
Classified as:
• Non-neoplastic
o Hamartomatous – glandular epithelium supported by smooth muscle cells that is contiguous with
the musularis mucosa – associated with Peutx-Jeghers syndrome
o Serrated (neoplastic or non-neoplastic)
o Submucosal (neoplastic or non-neoplastic)
• Neoplastic – adenoma or carcinoma
Management
• Polypectomy during colonoscopy
Surveillance
• If only 1-2 adenomas ≤10mm on baseline colonoscopy = lower risk – follow-up colonoscopy in 5 years
• If advanced adenomas (≥10mm, villous histology or high-grade dysplasia) or multiple adenomas (between
3-10) at baseline colonoscopy = higher risk – follow-up colonoscopy in 3 years
• 3-6 month and 12 months if sessile and laterally spreading
This 22 year old female patient presents with vague intermittent right sided abdominal discomfort
and some mild diarrhoea. What abnormality is evident here in the terminal Ileum and right colon at
colonoscopy?
What histopathological findings are anticipated in the biopsy findings of this lesion? What are possible
diagnoses?
?Likely Crohn’s:
Abnormality • Cobblestoning • Linear ulcers • Granulomas • Stricture
Likely histology: • Transmural inflammation- lymphoid aggregates • Granulomas - nonnectrotising • Cobblestoning • Linear ulcers/fissures • Skip lesions • Submucosal fibrosis
Possible diagnoses: • Crohn's disease • Ulcerative Colitis with backwash ileitis if pancolitis • NSAID enteropathy • Vasculitis • Eosinophilic gastroenteritis • Sarcoidosis
This lesion is found at colonoscopy in the sigmoid colon of an 85 year old male admitted acutely
with a near complete large bowel obstruction.
What is the lesion?
What initial treatment is most appropriate for this patient whose CT scan shows a large number of bilobar hepatic metastases?
What are the major risks of this approach?
CRC
Initial treatment
If the metastases are potentially resectable for cure, then an aggressive surgical approach is warranted for
both the primary and metastatic sites with the aim of curing the patient. If there are five or more
simultaneous, potentially resectable hepatic metastases (unless all are located in one
lobe), bilobar involvement, or if disease is borderline resectable due to location, initial chemotherapy
followed by reassessment and delayed resection is probably a better strategy than upfront surgery.
There are two main options for this bowel lesion. Acute resection or stenting to decompress the bowel
Jessica Redmond 90 followed by resection if deemed appropriate for the patient at a later stage. I believe the best option is stenting. Stenting is largely palliative Major risks of approach: Risks of neoadjuvant chemo: -failure to work -toxicity -risk of becoming unfit for surgery -
Risks of liver resection: • haemorrhage • Intra-abdominal sepsis • Liver failure • Bile leak • Liver steatosis • Mortality Risks of stenting: - perforation - failure to decompress - tumour ingrowth and re-obstruction - stent migration - bleeding - risks of anaesthetic -------- This patient's bowel obstruction needs to be managed. This 85 yo man may not be able to survive surgery, and metastases means most likely need palliative intervention. To manage the bowel obstruction: There are two main options for this bowel lesion. Acute resection or stenting to decompress the bowel followed by resection if deemed appropriate for the patient at a later stage. I believe the best option is stenting. Risks of stenting: - perforation - failure to decompress - tumour ingrowth and re-obstruction - stent migration - bleeding - risks of anaesthetic
With integration of surgery and chemotherapy, long-term survival can be achieved in as many as 50
percent of cases. For patients with borderline resectable metastatic disease, downstaging with neoadjuvant
chemotherapy may permit later successful resection. Even patients who are not candidates for a curative
resection can benefit from surgical palliation for symptoms of obstruction and bleeding from the
primary tumor.
4 rules for considering liver metastases resectable:
1) primary cancer is resectable,
Jessica Redmond 91
2) no extra-hepatic metastases,
3) at least 2 of 8 liver segments will remain preserving adequate hepatic function,
4) no involvement of major structures (portal vein, hepatic artery)
There is the possibility of neoadjuvant chemotherapy to allow for stage resection of liver metastases,
however, it is likely that if there is not enough preserved hepatic function from resection, than this would
not be possible, and definitely not curative.
On the other hand, if there are five or more simultaneous potentially resectable hepatic metastases (unless
all are located in one lobe), bilobar involvement, or if disease is borderline resectable due to location, initial
chemotherapy followed by reassessment and delayed resection is probably a better strategy.
As such, the focus would be on palliative control as opposed to curative control. Although this decision
likely needs to be made post-neoadjuvant chemotherapy
If 2 stage metastases resection:
There are two main options for this bowel lesion. Acute resection or stenting to decompress the bowel
followed by resection if deemed appropriate for the patient at a later stage. I believe the best option is
stenting.
Can do palliation
What is the obvious abnormality on this X-ray and in this operative photo?
In the presence of symptoms of large bowel obstruction what initial treatment options exist?
What can be done to prevent recurrence?
?volvulus Initial treatment of large bowel obstruction • Flexible sigmoidoscopy to detorse twisted segment • Rectal tube Prevent recurrence: • Endoscopic detorsion If acutely unwell: • O2 • IV fluids • Blood transfusions if anaemic • NG decompression • Pre-operative Abx • If perf- laporatomy
Volvulus? – Mx: Endoscopic decompression of sigmoid volvulus followed by elective resection for sigmoid
volvulus because of high rate or recurrence (up to 50%) with endoscopic decompression alone
• Urgent intervention needed due to risk of perf
• Rectal tube in situ for 24 hours with repeat AXR
• Endoscopic detorsion
• ?resection of sigmoid to prevent recurrence
• the literature indicates that sigmoid resection, done either as a Hartmann’s procedure when a
gangrenous colon is discovered at laparotomy or as a one-stage resection with primary anastomosis in
the setting of a viable colon, has the lowest rate of recurrence
Colorectal tumour? – Mx: Stenting
• May be ‘bridge to surgery’ i.e. provide time to correct fluid and electrolyte balance prior to operation or be
palliative
This is a photo of a patient with a loop de-functioning colostomy. What is the abnormality seen here?
What is the mechanism?
What treatment options are there?
Parastomal Hernia
Most frequent complication - 50% pts
Type of incisional hernia.
Stoma creation introduces an abdominal wall defect, the trephine, for which no healing is expected.
A parastomal hernia forms as the trephine is continually stretched by the forces tangential to its circumference.
Mx:
Asymptomatic: conservative management
Prosthetic mesh repair for acute obstruction or elective repair
Mucocutaneous separation
Separation of stoma from peristomal skin.
Leakage and skin irritation.
Mx:
Surgery
Stomal Necrosis
Early post operative period- venous congestion or arterial insufficiency from tight fascial opening.
Mx: Surgery
Dermatitis
Exposure to damaging effluent can produce severe skin breakdown in the area exposed to the drainage; this type of skin damage is characterized by severely denuded skin along the inferior aspect of the stoma and is most common among ileostomy patients and patients with problematic stomas.
Mx: Steroids
Stomal retraction
Stomal retraction is defined as a stoma that is 0.5 cm or more below the skin surface within six weeks of construction, typically as a result of tension on the stoma.
Mx:
Use of a convex pouching system and the use of a belt or binder. If these measures fail to provide a secure pouch seal, surgical revision may be needed.
Stomal Prolapse
Prolapse is the telescoping of the intestine out from the stoma and can occur with any type of stoma. Large abdominal trephine, increased intraabdominal pressure, and a redundant loop of bowel proximal to the stoma.
Mx:
Mannaged conservatively with cool compress and/or application of an osmotic agent (eg, table sugar or
honey) to reduce edema, followed by manual reduction of the prolapse and application of a binder with a prolapse over-belt to keep the bowel recued into the abdomen, or by pouching modifications to accommodate the prolapsed bowel when reduction
cannot be established or maintained.
What is the lesion visible here at colonoscopy and what has been done?
What is the possible histology?
What additional endoscopic manoeuvre is needed before completion of the colonoscopy?
?polyp - polypectomy
Histology- adenoma, adenocarcinoma
?adenoma – by definition histology is dysplasia
• Most colorectal cancers are caused by adenomas but only 5% of adenomas progress to cancer
• Development of adenoma to cancer usually takes 7-10 years
• Advanced adenoma is any adenoma with high-grade dysplasia, size >10mm, an adenoma with a villous
component
• Adenomas can be classified as tubular, villous or tubulovillous
• Mx: Polypectomy via snare excision
This patient presents with anal discharge and pain.
What is the likely nature of this lesion?
What does the initial management involve?
How is it usually treated?
What is the role of surgery?
Anal cancer
The lesions is likely a squamous cell carcinoma
Staging consists of biopsy of the primary tumour, CT chest abdomen and pelvis and PET
scans.
Chemoradiotherapy is the first line treatment, with radiation therapy administered to the
sites of the primary disease and lymphatic spread, usually with prophylactic irradiation of
clinically negative groins.
Local excision is an option for patients with T1 tumours less than 1 cm in size with followup
prophylactic chemoradiotherapy. Surgery also has a place in recurrent or persistent
disease after chemoradiotherapy. Although prognosis is poor abdominoperineal resection
offers the potential for long term survival.
NB- tumours above the dentate line, similar to rectal cancers, drain to the mesorectal and
internal iliac nodes whereas tumours below the dentate line may also spread to the
superficial inguinal and external iliac nodes.
Note: May also be a fistula. Initial management is surgery, and the mainstay of treatment is a fistulotomy.
This patient presented with severe anal pain after lifting a heavy piece of furniture. He feels this
perianal swelling but does not present to his GP for a couple of days.
What is the diagnosis?
What is the treatment?
Prolapsed internal haemorrhoid.
Management:
Medical
Grade 1 hydrocortisone cream short term for pruritis and pain
Anaesthetics- benzocaine rectal ointment
Stool softeners to decrease straining
Antispasmodics
Surgical:
Sclerotherapy- injection into the submucosal tissue of sclerosing agent with proctoscope to
obliterate the haemorrhoidal vessels
Banding- Cone around the base of the haemorrhoids to constrict the vessels
Haemorrhoidectomy- Indications include 3rd degree haemorrhoids. The mass is excised
with the overlying skin and mucosa, followed by secondary intention healing or staples for
larger haemorrhoids.
NB- external haemorrhoids are painful whereas internal haemorrhoids are not painful
This man had had an abscess drained in the perianal region.
Q1: What is the common presentation of this problem?
Q2: What is the first line of management?
Q3: What bacteria are usually cultured?
Q4: What is the usual cause of this?
Q1: What is the common presentation of this problem?
Constant and severe perianal pain and local tenderness
Q2: What is the first line of management?
Surgical incision and drainage
Q3: What bacteria are usually cultured?
Staphylococcus or E.coli
Q4: What is the usual cause of this?
Usually begin as acute purulent infections of the anal glands in intersphincteric spaces and
obstruct the ducts.
Obstruction- constipation, trauma or inflammation (Crohn’s disease or UC)
This patient had had an endoscopic mucosal resection (EMR) of a colonic polyp.
- What are the complications of this procedure?
- What is the likely Pathology here?
- What would normally occur if there is found to be a tiny focus of completely excised, well differentiated invasive cancer in the specimen that is far from the
resection line?
- What are the complications of this procedure?
Bleeding- 2-11%
Perforation
Failure to completely resect polyp - What is the likely Pathology here?
Adenoma - What would normally occur if there is found to be a tiny focus of completely excised, well differentiated invasive cancer in the specimen that is far from the
resection line?
Frequent surveillance.
If advanced adenomas (>1cm or high grade dysplasia) or multiple adenomas (between 3 and 10) at baseline
colonoscopy = high risk = colonoscopy in 3 years.
12 month if sessile and laterally spreading
This is a photo of the anal canal taken at colonoscopy by retroflexion of the scope.
Q1. What does this show?
Q2. How do these present clinically?
Q3. How are they treated?
Q1. What does this show? ?haemorrhoids Anal rectal carcinoma Fissure polyp Anal cancers
Q2. How do these present clinically?
Haematochezia (45% of patients), anal pain, sensation of an anal mass, pruritis, anal
discharge, on DRE they may have a localised firm or hard ulcer/ growth with an irregular
surface and edge, possibly surrounded by woody induration.
Q3. How are they treated?
The lesion is confirmed by biopsy
1. Chemoradiotherapy
2. Abdominoperineal resection
This patient presents with perianal itching.
Q1. What are the lesions you can see in the perianal area?
Q2. What is the responsible agent that causes this condition?
Q3. What treatments are available?
Q1. What are the lesions you can see in the perianal area?
Anogenital warts/ perianal warts/ condylomata acuminata
Other causes:
- Faecal soilage
- Herpes
- Mucous leakage
- Diarrhoea
- Haemorrhoids
- Skin disorders - contact dermatitis from soaps, shower gel, cream, talc etc
- Fungal infections (candida)
- Genital warts:
- > Chlamydia trachomatis, syphillis, gonorrhoea
Q2. What is the responsible agent that causes this condition?
HPV. Types 6 and 11 are normally the causative agents of anal warts. Transmitted sexually.
Q3. What treatments are available?
All patients should be offered treatment although although warts may resolve.
Topical agents- podophyllin (anti-mitotic)
Trichloracetic acid (caustic causing chemical coagulation of protein) Immunomodulators such as imiquimod
Cryotherapy
Surgical excision under local or general anaesthetic (only practical option when large numbers are present). Care must be taken to prevent the complication of anal
stenosis.